Sie sind auf Seite 1von 75

PN NCLEX INTEGRATED A1

Practice Test for PN - NCLEX:

1. A mother reported that her son is throwing up each time she feeds him. What would be the best
assessment question the nurse would ask?
a. Did you warm up the formula?
b. What kind of formula did you give him?
c. Does your son feel hungry each time he throws up?
d. Does your son have a jelly-like stool?
Answer: D – jelly-like stool is an indication of GI problem.

2. Rosemarie is 24 years old, G1P0, admitted with a diagnosis of Multiple Sclerosis. The nurse
observes indications of positive response from treatments and signs that the patient is recovering
from the disease by which of the following assessment data?
a. Less frequent mood swings.
b. Able to wiggle toes and move fingers.
c. Worsening urinary incontinence.
d. Ambulates with a more stable gait.
Answer: D – Multiple Sclerosis temporarily affects both sensory and motor. Being able to walk with stable
gait is a sign that the motor function is returning to normal.

3. Rolland works in the construction site and sustains head injury when a small piece of metal dropped
on his head. As a result of the accident, he experiences brief loss of conciousness. He was admitted in
the medical unit for further investigation and sub dural hematoma is suspected. Important nursing
care provided for Mr. T includes:
a. Side lying
b. Semi fowler’s
c. High fowler’s
d. Flat
Answer: B – due to possible increased ICP, semi-fowlers’ position is appropriate to promote venous
drainage and decreases ICP.

4. Ron is 4-year-old child being discharge to home after an uneventful tonsillectomy. The nurse would
suggest that the appropriate car seat for the child is
a. the rear-facing seat.
b. the front facing seat.
c. the booster seat with car seat belt.
d. the back seat.
Answer: C – booster car seat is appropriate for a bigger child.

5. Mr. Dubin is admitted with a diagnosis of severe anemia. He was ordered two units of packed cells.
Should Mr. Dubin develops hemolytic transfusion reaction, nurse will immediately stop the
transfusion and then
a. run the IV NS.
b. send the remaining blood to the lab.
c. monitor the vital signs.
d. collect urine specimen to be sent to the lab..
Answer: A – Normal Saline is compatible with blood transfusion. The physician must be informed after
running the NS.

6. A patient with rheumatoid arthritis complains to the nurse about her back pain and said, “I want a bed
that is similar to what I have at home.” What would be an appropriate nursing response?
a. “I will massage your back to ease up the pain.”
b. “I will ask the physician to order analgesic for you.”
c. “I will refer you to the occupational therapist.”
d. Ask the husband how the patient sleeps at home.
Answer: D – assessment is the first step to be taken by the nurse.

7. Bonnie and 4-year-old daughter moved to a new apartment. Two days after the move the girl became
ill and was taken to the ER. Lead poisoning was suspected (plumbism). Lead was possibly inhaled
from the fresh paint in the apartment. The nurse is aware that lead can fatally affect the
a. heart.
b. brain.
c. skin.
d. kidney.
Answer: B – lead is inhaled and deposited in bone marrow and blocks the formation of RBC but it can
seriously affect the CNS.

8. After a craniotomy for the removal of hematoma sustained in a fall, the patient is returned to the
recovery room. The nurse places the child in a semi-fowler’s position to
a. increase venous drainage and prevent accumulation of fluid.
b. promote lung expansion and improve oxygenation.
c. decrease pressure on the diaphragm and increase thoracic pressure.
d. promote oxygenation and keeps the child more alert..
Answer: A –this position utilizes gravity to drain fluid from the head and prevent fluid accumulation.

Case Study:

Francis is a 48-year-old female, patient transferred in from a community hospital admitted there on 2/4/06
with lower abdominal pain, which shows free air on X-ray. For few days prior to admission, she was
taking Tylenol #3 for abdominal and feet pain which causes her constipation. Her past medical history
includes blindness both eyes; IDDM since age 8; failing cadaveric kidney —had renal transplant in ’88;
developed skin cancer from Cyclosporin; has HTN, Cholesterol, CAD-MI in ’98; PVD-bilateral feet ulcer;
recurrent UTI’s. In May 2001, she went for R forearm AV fistula placement. However, due to uncontrolled
HTN, the AV fistula has a very weak bruit, which cannot be used for hemodialysis. Due to intestinal
obstruction (from Tylenol #3—constipation), on April 12, 2002, she had resection of the sigmoid and a
temporary colostomy was put in.

Qestions

9. During admission, Francis’ blood sugar was noted to be 15 mmos/L. Important treatment for Francis
would include:
a. IV fluids
b. Potassium
c. NPH insulin
d. Kayexalate
Answer: A – IV fluids are given to combat dehydration. Glucose has an osmotic diuretic effect causing
polyuria.

10. An immediate danger to confront Francis secondary to her elevated blood sugar level is
a. albumin is negative.
b. dehydration and impending shock.
c. hyperglycemia.
d. renal failure.
Answer: B – glucose has an osmotic diuretic effect causing polyuria.

15. The type of insulin expected to be ordered by the physician to treat Francis elevated blood sugar
would be:
a. NPH
b. Regular
c. Protamine Zinc
d. Globulin
Answer: B - Short acting or regular insulin usually reaches the blood within 30 minutes after injection,
peaks in 2 to 4 hours and stays in the bloodstream for 6 to 8 hours. Because of its short and predictable
peak action, regular insulin is the insulin routinely used for intravenous administration.

16. Francis went into diabetic coma. The nurse may observe which one as typical manifestation (s) of
ketoacidosis:
a. Slow, bounding pulse.
b. Deep, rapid respirations.
c. Diaphoresis, cold clammy skin.
d. Dilated pupil, hypotension.
Answer: B—The body’s effort to get rid of the acid through breathing.

17. Francis’ serum K⁺ level is 3.8 mEq/L. Potassium chloride 40 mEq has been prescribed to be added
to the first liter of IV fluid. The nurse should
a. Question the physician’s order because the K⁺ level is within the normal limit.
b. Question the physician’s order because the K⁺ level is above the normal range.
c. Recognize that potassium is transferred into the cellular compartment during insulin therapy.
d. Recognize that potassium is being given to correct the existing hyponatremia.
Answer: C—ketoacidosis means high blood glucose. Insulin is expected to be administered to lower the
blood glucose level. At the same token, insulin transport potassium back to the cell. Thus, potassium IV is
usually given prophylactically with insulin.

18. The PN assigned to Francis knows the importance of rotating insulin injection sites. Injection
should be given in any one spot more often than every two weeks. Site rotation is important to
prevent or combat which of the following complications?
a. Abnormal tissue changes which may result in poor absorption of the injected insulin.
b. Rapid absorption of insulin due to tissue sensitivity.
c. Formation of glucagon promotes a rise in blood sugar, thereby, producing hyperglycemia.
d. Increased blood glucocorticoids raise blood sugar by stimulating the conversion of fat and
protein into glucose.
Answer: A -- To avoid tissue atropy or hypertrophy, patients should be taught to avoid injecting insulin in
the same site each time. Instead, they should be advised to give the injection in the same general area. For
example, giving a "before breakfast" injection in the abdomen and "before supper" injection in the leg each
day will give similar blood glucose results.

19. Which of the following drugs prescribed to Francis contributes to hyperglycemia?


a. Prednisone
b. Zantac
c. Tylenol #3
d. Antibiotic

Answer: A – Prednisone is steroid that increases blood sugar.

20. The importance of monitoring blood sugar, activity level, and insulin doses is to
a. adjust the activity level based on the blood sugar level.
b. adjust the diet and insulin doses as the activity level increases or decreases.
c. adjust the diet and activity level based on the blood sugar level.
d. adjust the insulin doses based on the sugar level in the urine.
Answer: B- If a patient injects too much insulin, exercises too much in relation to carbohydrate consumed,
or does not eat enough or eat at the appropriate times, the blood glucose level can fall low enough to cause
hypoglycemia.
23. Insulin or oral hypoglycemic agents, diet and exercise are the most effective ways in treating diabetes.
Oral hypoglycemic agents may be used for patients with:
a. Ketosis
b. Obesity
c. Type I DM
d. Some insulin production
Answer: D – oral hypoglycemics may be helpful when some functioning of the beta cells exists, as in type
II DM.

24. The best indication that a patient with diabetes mellitus is successfully managing the disease after
discharge is a
a. significant lost of body weight.
b. well controlled serum glucose.
c. steady loss of weight.
d. insulin orders are being followed.
Answer: B- a combination of diet, exercise, and medication is necessary to control the disease; the
interaction of these therapies is reflected by the serum glucose.

25. Which of the following clients will most likely develop Diabetes Mellitus?
a. American Indian
b. Client with grandparents have diabetes
c. Asian
d. European
Answer: B—Diabetes is hereditary.

Case Study:

Antoinette is 47- year- old woman with history of hypertension presented to ER with complain of sudden
pain in her heel. Her lifestyle is sedentary and denies history of trauma or rigorous physical exertion. On
admission assessment, spontaneous rupture of the Achilles tendon was diagnosed and treatment was
initiated.

Few weeks following her ER visit, Antoinette sought the help of a family physician for prescription of pain
medication. Her physical assessment showed central obesity, thin musculature, and abdominal striae.
Cushing's syndrome was suspected. Further endocrine assessment showed clinical features typical of
Cushing's syndrome, which had been present for at least four years. Investigations confirmed Cushing's
syndrome as a result of an adrenal adenoma.

Questions

26. The PN who performs the admission assessment on Antoinette would expect to observe which of the
following?
a. Hair growth in the face and extremities.
b. Taut, dry skin.
c. Thin, pale appearance.
d. Protruding eyes.
Answer: A – excessive hair growth in the woman’s face and extremities caused by excessive
glucorticoid hormones.

27. Antoinette may also manifest signs of diabetes mellitus because:


a. The ACTH causes hypermetabolism causing rapid weight loss.
b. Excessive ACTH secretion alters the secretion of insulin
c. The abnormal insulin production results to catabolism.
d. The excessive glucocorticoids secreted by the adrenal gland caused excessive
tissue catabolism and gluconeogenesis.
Answer: D – excess glucocorticoids cause hyperglycemia and signs of DM may develop.

28. Additional assessment on Antoinette include


a. dehydration and menorrhagia.
b. buffalo hump and hypertension.
c. pitting edema and frequent colds.
d. migraine headache and dymennorhea.
Answer: B – Cushing’s syndrome results from excess adrenortical activity. Signs include slow wound
healing, buffalo hump, hirsutism, weight gain, hypertension, acne, moon face, thin arms and legs, and
behavioral changes

30. Due to excessive amount of glucocorticoids, Antoinette will have which of the following problems?
a. Urine output
b. Glucose level
c. Sodium retention
d. Serum potassium
Answer: B – as a result of increased cortisol levels, patients experience increased blood glucose.

31. Following a head injury a patient develops a deficiency in anti diuretic hormone (ADH). Normally
secretion of ADH
a. prevents diuresis.
b. promotes diuresis.
c. causes hypoperfusion of kidney tissues.
d. promotes retention of potassium.
Answer: A – reabsorption of sodium and water in the tubules decreases urinary output and retains body
fluids.

Case Study:

Valerie, 37 years old, enters the hospital with symptoms of Addison’s disease. She relates that she was
taking cortisone pill for her contact dermatitis, but they bothered her stomach so she quit taking them.

Questions

32. On admission Valerie’s vital signs are: BP was 80/60; HR 115; RR 24. Laboratory results include a
sodium level 128 mEq/L; potassium 5.5 mEq/L. Hypotension in Addison’s disease involves a
disturbance in the production of
a. Cortisol
b. Androgen
c. Glucocorticoid
d. Mineralcorticoid
Answer: D – mineralcorticoids such as aldolterone cause the kidneys to retain sodium ions. With sodium,
water is also retained, elevating blood pressure. Absence of this hormone thus causes hypotension.

33. The nurse should observe Valerie closely for signs of infectious complications because in Addison’s
disease there is disturbance in
a. fluid balance.
b. oxygenation.
c. electrolyte balance.
d. inflammatory process.
Answer: D – because of diminished glucorticoid production, there is a decreased inflammatory effect.

34. On physical assessment, nursing notes describe Valerie as emaciated, with thin musculature, weak and
easily fatigue. These findings are primarily due to
a. hypovolemia; decreased glucocorticoids.
b. hypervolemia; increased glucocorticoids.
c. increased cortisol level; increased blood glucose.
d. hypernatremia; hypokalemia.
Answer: A – there is decreased mineralcorticoid (aldosterone) causing hypovolemia and decreased
glucocorticoid (altered metabolism) in Addison’s disease.

35. The physician orders high protein, high calorie and high salt diet on Valerie. When ask by the patient,
the nurse would explain that
a. high salt diet replaces potassium loss in the urine.
b. high protein, high calorie diet is needed for tissue repair and energy
expenditure.
c. high protein and high calorie diet is important in gaining weight .
d. extra salt is needed to increase the low sodium level in Addison’s disease.
Answer: D – lack of mineralcorticoids (aldosterone) leads to a loss of sodium ions in the urine and
subsequent hyponatremia.

36. Therapy for Valerie who has Addison’s disease is aimed chiefly at
a. dietary supplements to increase the potassium level.
b. dietary restrictions to decrease the blood sugar level.
c. maintaining fluid and electrolyte balance.
d. insulin administration to control the blood sugar.
Answer: C- lack of mineralcorticoids causes hyponatremia, hypovoloemia, and hyperkalemia. Dietary
modification, as well as administration of cortical hormones, is aimed at correcting these electrolyte
imbalances.

37. Which of the following conditions indicate adrenal insufficiency in a critically ill patient?
a. Hypoglycemia and hyperkalemia.
b. Hypernatremia and hyperglycemia.
c. Hyperkalemia and elevated cortisol level.
d. Hyponatremia and elevated cortisol level.
Answer: A – due to decreased cortisol (anti-insulin) and decreased aldosterone (sodium) levels in Addison’s
disease, the patient experiences hypoglycemia.

Case Study

Ms. Wilson is a 45-year-old female presents to the clinic with unexplained weight loss, anxiety, heat
intolerance and palpitations. Exams reveal mild systolic hypertension, exophthalmos, lid lag, non-pitting
leg edema, and a diffusely enlarged painless thyroid gland. Lab results showed depressed TSH and
borderline T3 and T4. Hyperthyroidism was confirmed.

Questions

38. As a result of low levels of T3 and T4, the nurse should expect Ms. Wilson to exhibit:
a. Irritability
b. Tachycardia
c. Cold intolerance
d. Profuse diaphoresis
Answer: C- decreased production of thyroid hormones lowers metabolism, which leads intolerance to cold.

40. Ms. Wilson was admitted in the hospital for subtotal thyroidectomy. Two days prior to surgery she
complains to the nurse and states, “It looks like that she is coming down with flu."An appropriate
nursing action is:
a. Hold the Lugol’s solution and inform the physician.
b. Monitor the vital signs and report the temperature to the physician.
c. Obtain blood specimen for culture and send it the lab.
d. Monitor the patient very closely and report any abnormal findings to the physician.
Answer: A – Lugol’s solution is a medication given few days before thyroidectomy to decrease the
vascularity of the thyroid glands. Should the patient develop infection, thyroidectomy will be cancelled,
thus, the Lugol’s solution should not be given.

41. Which of these assessment data on Ms. Wilson requires quick intervention by the nurse?
a. “I eat a lot and unable to control my appetite.”
b. “I can hear my heart beating faster and feel dizzy.”
c. “I feel hot all the time.”
d. “I became very impatient lately.”
Answer: B – Tachycardia is a serious complication of hyperthyroidism. This must be reported to the
physician ASAP for prompt treatment.

42. On the day of surgery, the nurse noted the following laboratory results. Which one should be reported
to the surgeon immediately?
a. Creatinine level 1.7 mg/dl.
b. WBC 12,000.
c. Hemoglobin 12 mg/dl.
d. Potassium level 5.0 mEq/L.
Answer: B – elevated WBC is a sign of infection.

43. Ms. Wilson has undergone total thyroidectomy. Following surgery, the patient should be observed
for signs and symptoms of thyroid storm. Which of the following is consistent with thyroid storm?
a. Heart rate of 180 bpm.
b. Subnormal body temperature.
c. Diarrhea.
d. Irritability.
Answer: A – Tachycardia is one of the manifestations of thyroid storm.

44. Which symptom listed below would indicate a complication following thyroidectomy?
a. Sore throat.
b. Difficulty in flexion and hyperextension of the neck.
c. Muscle cramping.
d. Initial swallowing difficulties.
Answer: C—due to hypocalcemia (complication of thyroidectomy) the patient will experience muscle
cramping.

45. Which of the following symptoms listed below would indicate laryngeal nerve injury following
thyroidectomy?
a. Hoarseness.
b. Numbness and tingling of fingers and toes.
c. Painful spasms.
d. Facial muscle twitch.
Answer: A – Hoarseness is a sign of laryngeal damage.

46. To evaluate possible laryngeal nerve injury following a thyroidectomy, the nurse on an hourly basis
should:
a. Ask the patient to speak.
b. Ask the patient to swallow.
c. Have the patient hum a familiar tune.
d. Swab the patient’s throat to test gag reflex.
Answer: A - if the laryngeal nerves are injured bilaterally during surgery, the vocal cords will tighten,
interfering with speech. If one cord is affected, hoarseness develops. This can be evaluated simply by
having the patient speak every hour.

47. An accidental removal of the parathyroid glands during a thyroidectomy would cause:
a. Myxedema
b. Tetany and death
c. Hypovolemic shock
d. Adrenocortical stimulation
Answer: B- parathyroid removal eliminates the body’s source of parathyroid hormone, which increases
blood calcium. The resulting low body fluid calcium affects muscles, including the diaphragm, resulting in
dypsnea, asphyxia, and death.

48. When a patient return from the recovery room following a subtotal thyroidectomy, the nurse should
immediately
a. inspect the incision.
b. instruct the patient not to speak.
c. keep the patient supine for 24 hours.
d. place a tracheostomy set at the bedside.
Answer: D – thyroid surgery sometimes results in accidental removal of the parathyroid glands. A resultant
hypocalcemia may lead to contraction of the glottis, causing airway obstruction; edema also causes
obstruction.

49. When changing Ms. Wilson’s postoperative dressing, the nurse is careful not to introduce
microorganisms into the surgical incision. This is an example of:
a. Wound asepsis
b. Medical asepsis
c. Surgical asepsis
d. Concurrent asepsis
Answer: C – intact skin is the first line of defense against entry of microorganisms. A surgical incision is a
portal of entry, so a technique that requires the absence of all microorganisms (surgical asepsis) is essential.

50. The hormone that tends to decrease calcium concentration in the blood is:
a. Calcitonin
b. Aldosterone
c. Thyroid hormone
d. Parathyroid hormone
Answer: A – calcitonin, a thyroid gland hormone, prevents the reabsorption of calcium by bone. It also
inhibits the release of calcium from bone. The net result is lowered serum calcium levels.

51. When assessing for complications of hyperparathyroidism, the nurse should monitor the patient for:
a. Tetany
b. Seizures
c. Grave’s disease
d. Bone destruction
Answer: D – hyperparathyroidism causes calcium release from the bones, leaving them porous and weak.

52. When giving health and safety hazards on a mother with a 3-month-old infant, the nurse should
emphasize that
a. tiny objects must be removed from crib.
b. cover electric outlets with the safety plugs.
c. gates on doors must be installed.
d. close the washroom door at all times.
Answer: A – by 4 months of age infants picks anything within reach and put right through the mouth.

53. The nurse should observe a child with acute glomerulonephritis primarily for:
a. Hematuria; hypotention.
b. Diuresis; dehydration.
c. Hypertension; hematuria.
d. Poor appetite; poor skin turgor.
Answer: C – the decreased filtration of plasma in the glomeruli results in an excess accumulation of water
and sodium. Hypertension is thought to be due to hypervolemia, although its exact cause is unclear.
54. Which of the following is considered a sequela of a staphylococcal infection that may result to
glomerulonephritis?
a. Infected burn wound
b. Impetigo
c. Skin problem from chickenpox
d. Herpes simplex
Answer: B-impetigo is a bacterial infection of the skin caused by streptococci or staphylococci. Group A
hemolytic streptococci can cause rheumatic fever and glomerulonephritis.

55. Baby Larry was born with Down’s syndrome, when performing physical assessment, the nurse should
carefully evaluate the infant’s:
a. Heart sounds
b. Gluteal fold
c. Lung expansion
d. Normal reflexes
Answer: A – cardiac anomalies often accompany other genetic problems such as Down syndrome. 30 to
40% of these infants have congenital heart defects.

56. While working on his motor bike, a 19-year-old sustains a puncture wound from a rusty metal. He
presents himself to the ER of local hospital. The puncture wound is cleansed with hydrogen peroxide
and a sterile dressing applied. The client confirmed that he is up-to-date with his tetanus
immunization. Antibiotic was prescribed and administered and the client was sent home with
instructions to return if there is any change in the wound area. Three days following the ER visit, the
client is admitted to the hospital with a diagnosis of tetanus. Nursing care for client includes:
a. Admit the client in a private room.
b. Admit the patient in a two-bedded room.
c. Monitor the patient’s vital signs every two hours.
d. Encourage the patient to move from side to side every hour.
Answer: A –slightest stimulation sets off a wave of very severe and very painful muscle spasms involving
the whole body. Nerve impulses cross the meyoneural junction and stimulate muscle contraction due to the
presence of exotoxins produced by Clostridium tetani.

57. Congenital hip dysplasia is suspected when the assessment data indicates which of the following in the
newborn?
a. Presence of Ortalani’s click
b. Symmetry of the gluteal folds.
c. Both legs abducted well.
e. Both legs are the same in length.
Answer: A- the presence of Ortalani’s click when legs are abducted and asymmetry of gluteal folds are
signs of congenital hip dysplasia

58. While caring for a young child in Bryant’s traction, the nurse should be aware that the traction is no
longer effective if:
a. the child cannot move his leg in traction.
b. the child’s leg in traction is raised on the pillow.
c. the child’s hips are off the bed.
d. the child’s hips touch the bed.
Answer: D – the principle of Bryant’s traction is bilateral 90° hip flexion. It is skin traction applied to the
legs to decrease the fracture, maintain alignment, and immobilize both legs.

59. A 15-year-old male is admitted with partial and full-thickness burns of the arms and upper torso. The
nurse plans for the administration of pain medication intravenously rather than intramuscularly because
this method of administration:
a. Decreases the risk of tissue irritation.
b. Reduces severe pain more effectively.
c. Bypasses impaired peripheral circulation.
d. Provides for more prolonged relief of pain.
Answer: C-damage to tissues interferes with stability of peripheral circulation, precluding the use of IM
medications.

Case Study

B.C. is a 14-yr-old boy who was brought to the ER after he had sustained flame burns. He had been
wearing masquerade costume during Halloween celebration when his costume was set ablaze by the lighted
candles in the party room. He extinguished the fire by rolling on the floor. On admission, resuscitation was
commenced with intravenous fluids, antibiotics, and wound dressing.

On examination he was found to be a young man, not pale, but febrile (body temperature, 37.8 °C). He was
tachypneic, RR 26 per min, with audible breath sounds bilaterally.

Burn assessment revealed a total body surface area involved of 30%, with mixed-thickness burns sparing
some patchy areas of the back and the right upper limb.

Questions

60. B.C. anterior chest was affected. For the first 24 hours after hospitalization, the nurse should
primarily
observe him for
a. wound sepsis.
b. complain of pain.
c. dyspnea or stridor.
d. initiate IV therapy.
Answer: C- Inhalation burns are usually present with facial burns, regardless of the depth; the immediate
threat to life is asphyxia from irritation and edema of the respiratory passages and lungs.

61. Fluid shift occurs immediately following the onset of burn. Nursing assessment indicating fluid shift
include:
a. Rising blood pressure.
b. Decreasing capillary refill.
c. Loss of sodium and increased potassium.
d. Irreversible shock 2 hours after burn.
Answer: B – delayed capillary refill is evident due to lack of volume and oxygen.

62. Calculation of fluid replacement in burn is based on the patient’s


a. Age
b. Height
c. Weight
d. Percentage of burn & weight
Rationale: C - calculation of fluid replacement in burn is based on weight and BSA burn (may use rule of
nine to calculate burn area)

63. Which of the following data reflects that fluid resuscitation on B.C was effective?
a. Urine output of 300 cc/24H; increased BUN, increased Na⁺ level.
b. Urine output of 400cc/24H, increased BUN, decreased Na⁺ level.
c. Urine output of 30/H, decreased BUN, decreased Na⁺ level.
d. Urine output of 30cc/H, decreased BUN, decreased K⁺ level.
Answer: C – all data reflect kidney perfusion.
64. During the second phase of burn recovery, the nurse anticipates that the client will exhibit
a. high serum potassium level.
b. an increased urinary output.
c. an elevated hematocrit level.
d. a fall in central venous pressure.
Answer: B—sign of adequate hydration.

65. B.C. belongs to the Mormon’s church. Which of the following snacks will be served?
a. Coffee and muffins.
b. Orange and wheat bread.
c. Ice cream and Cookies.
d. Coca-cola and pizza.
Answer: B—Mormons are very health conscious.

66. B.C underwent skin grafting. Vital signs are BP 124/68; HR 100 bpm; RR 24; T 37.7ºC, potential
complication that can be possibly developed will be:
a. Sepsis
b. Hypovolemic
c. Pain
d. Electrolyte imbalance
Answer: A – infection, sepsis, is a serious complication of skin grafting.

67. At age 3 months, an infant is admitted to the hospital for a cleft lip repair. Postoperative nursing care
should include:
a. Using spoon to administer oral feedings.
b. Cleansing the suture line to prevent infection.
c. Allowing the infant to suck on a pacifier to prevent crying.
d. Positioning the infant on an abdomen to avoid aspiration.
Answer: B-meticulous care of the suture line is necessary to prevent infection and to provide the best
cosmetic effect.

68. The type of stool in cystic fibrosis is described as foul-smelling, frothy called steatorrhea. This is
caused by presence of large amounts of:
a. Undigested fats
b. Blood
c. Undigested foods
d. Bilirubin
Answer: A-because of lack of the pancreatic enzyme lipase, fats remains unabsorbed and is excreted in
excessive amounts in the stool.

69. The child with PKU is encouraged to eat food such as


a. Meat and poultry.
b. Low-phenylaline diet.
c. Fruits and vegetables.
d. Eggs, dairy chicken.
Answer: B- in PKU the absence of hepatic enzyme phenylalanine hydroxylase prevents normal
metabolism (hydroxylation to tyrosine) of the amino acid phenilalanine in the body and the alternate
metabolic by-products (phenyketones) are associated with severe mental retardation (exact mechanism is
not known).

70. Forty eight to 72 hours post birth, blood test for PKU is taken on the neonate. This test is called
a. Tensilon test
b. Guthrie test
c. Schilling test
d. Sodium Chloride test
Answer: B- the Guthrie blood test reliably detects abnormal phenylalanine levels as early as 4 days of age,
provided the infant fed a milk diet.

71. The most effective time for the nurse to perform a cellophane-tape test for Pinworms is:
a. Before bedtime.
b. When the peristalsis is least active.
c. After taking shower.
d. Early morning upon arising.
Answer: D-the adult pinworm lives in the rectum or colon and emerges onto the perirectal skin during
hours of sleep; depositing her eggs during this time.

72. The nurse doing a newborn assessment counts the infant’s cord vessel. In a normal infant there are:
a. Two vessels: one vein and one artery.
b. Three vessels: two veins and one artery.
c. Three vessels: one vein and two arteries.
d. Four vessels: two veins and two arteries.
Answer: C-there are 3 vessels: one vein (carry oxygenated blood to the fetus) and two arteries (return
deoxygenated blood to the placenta).

73. Besides acute glomerulonephritis, a sequela of Streptococcus infection would be:


a. Nephritic syndrome
b. Tonsilitis
c. Rheumatic fever
d. Strep throat
Answer: C-rheumatic fever is an inflammatory disease involving the joints, heart, CNS, and subcutaneous
tissue. It is believed to be autoimmune process that causes connective tissue damage.

74. A child comes to the hospital after exposure to diphtheria and is given antitoxin. This type of
immunity is known as:
a. Active natural immunity.
b. Active artificial immunity.
c. Passive natural immunity.
d. Passive artificial immunity.
Answer: D- in passive artificial immunity, an antibody made is another organism is injected into the
infected or presumed infected person to provide immediate immunity to the invading organism.

75. Immunity by antibody formation during the course of a disease is:


a. Active natural immunity.
b. Active artificial immunity.
c. Passive natural immunity.
d. Passive artificial immunity.
Answer: A-in active natural immunity, the infected person’s immune system responds to the invading
organism by producing antibodies specific for the invader.

76. When reviewing the immunization schedule for a 12-month-old, the nurse would expect that the
infant had been previously immunized against:
a. Pertussis, tetanus, polio, and measles.
b. Polio, pertussis, tetanus, and diphtheria.
c. Measles, mumps, rubella, and tuberculosis.
d. Measles, rubella, polio, tuberculosis, and pertussis.
Answer: B-the recommended immunization schedule for infants is administration of the combined
diphtheria, pertussis, and tetanus vaccine and the trivalent oral polio virus at ages 2, 4 6 months.

77. Anemia, is a blood disease caused by a nutritional problem, blood loss, and bone depression. In
nutritional anemia, the patient is given food rich in protein, iron, vitamin B12 and:
a. Sodium
b. Potassium
c. Folic acid
d. Multivitamins
Answer: C-Folic acid acts as necessary coenzyme in the formation of heme, the iron-containing protein in
hemoglobin.

78. The incidence of Sickle Cell Anemia is higher among black American babies. The symptoms of
sickle cell anemia are not evident until later during infancy because
a. The baby is fed with milk formula, which is rich in iron.
b. The infant has a much higher RBC count than children and adult.
c. Maternal iron is depleted later in infancy.
d. Infants have more body fluids than any age group.
Answer: C-high levels of fetal hemoglobin prevent sickling of red blood cells. The newborn has from 44%
to 89% fetal hemoglobin, but this rapidly decreases during the first year.

79. A 25-year-old young male comes into the ER following a motorbike accident. Upon obtaining a
recent history, the patient states he fell in the ground and hit his head on the pavement. He reports
that he was unconscious for 15 seconds and then confused for several minutes. What is the first
diagnostic test to perform on this patient?
a. Blood glucose level
b. Skull X-ray
c. CT scan of the brain
d. Myelogram
Answer: C –CT scan must be performed as soon as possible to rule out a cerebral hemorrhage. Blurred
vision is considered a focal sign and when coupled with loss of consciousness may indicate a more severe
brain injury. Skull films are not necessary, as bones can be visualized on CT scan. Blood work and EEG
may be done later if indicated.

80. Mr. Smith is 67-year-old black male brought to the hospital by his wife, who stated that he fell down
20 minutes ago and has been unable to speak or move his right side since then, Mr. Smith has no
significant past medical history. On exam, Mr. Smith is conscious, very anxious, his speech is garbed
and unintelligible, he has a left facial droop, and he is completely right hemiphlegic. The most likely
etiology for his symptoms is:
a. CVA
b. Traumatic brain injury
c. Brain tumor
d. Alzheimer’s disease
Answer: A – Mr. Smith has all of the classic symptoms of a stroke or CVA. Although he did have a fall that
could lead the RN to suspect traumatic brain injury, rarely does a patient maintain consciousness following
an injury severe enough to cause Mr. Smith’s symptoms.

81. Given Mr. Smith’s clinical picture, which of the following would be the first diagnostic tests that
must be performed?
a. CT scan of the brain
b. MRI of the brain
c. Cerebral angiogram
d. EEG
Answer: A –given suspicion of a stroke, the first test to perform would be a CT scan to determine the
presence or absence of hemorrhage

82. Gout is more prevalent in which of the following:


a. Men
b. Women
c. Young adults
d. Asians
Answer: A – gout has a higher prevalence in middle aged or older men.
83. Mr. O, age 75, fell recently and sustained wrist fracture. What other disease would you suspect?
a. Osteoporosis
b. Lupus
c. Fibromyalgia
d. Rheumatoid Arthritis
Answer: A – the incidence of osteoporosis in men increases with age.

84. A diet rich in calcium over a lifetime will decrease the risk of
a. Heart disease
b. Osteoporosis
c. Gout
d. Rheumatoid Arthritis
Answer: B – a diet rich in calcium over lifetime will decrease the risk of osteoporosis.

85. Frequent urinary tract infection is associated with all of the following conditions except:
a. DM
b. Male sex
c. Indwelling catheter
d. Bladder dysfunction
Answer: B – the frequency of urinary tract infections is much lower among men, although it does increase
in older men with prostate hypertrophy.

86. You are asked to see Mr. J, a 45-year-old male admitted to the medical floor. He has a temperature of
103°F, complains of urgency, frequency, and perineal pain. He states his urine is somewhat cloudy in
appearance. Mr. K denies any trauma to the genital area. On physical assessment, Mr. J appears
acutely ill, flank tenderness is negative to percussion, abdominal assessment is unrevealing. Rectal
exam reveals a tender prostate. You anticipate all of the following to be ordered except:
a. CBC
b. UA
c. Urine culture
d. PSA (prostate specific antigen)
Answer: C – the patient’s symptoms are suggestive of infection; urine culture will confirm the infection.

87. Symptoms of BPH are primarily seen in men:


a. Age 18 – 25.
b. Age 26 – 35.
c. Age 36 – 40.
d. Age over 50.
Answer: D – BPH has unkown cause, however, it is more prevalent in men above 50 years of age.

88. A patient has been started on metformin for type 2 DM. You would tell your patient that the most
common side effects of this drug are:
a. Hypoglycemia
b. Headaches
c. Nausea and abdominal pain
d. Constipation
Answer: C – frequent side effects of metformin include nausea and abdominal discomfort. Liver and
kidney function tests are recommended before starting the patient on the drug and every 6 to 22 months
thereafter.

89. The initial treatment of choice for symptoms of palpitations, tachycardia, and tremor seen in thyroid
storm is:
a. Ativan
b. Levothyroxine
c. Radioactive iodine
d. Inderal
Answer: D – Inderal is a beta blocker which decreases heart rate.

90. Physical findings commonly seen in hypothyroidism include:


a. Coarse hair, thin brittle nails
b. Malnourished appearance, alopecia
c. Tachycardia, hyperreflexia
d. Confusion, stupor
Answer: A – these are signs of hypothyroidism.

91. Patient education for hypothyroidism would include instructing the patient that
a. After 2 months they may discontinue their thyroid medication.
b. Weekly blood tests will need to be done while on thyroid medication.
c. Thyroid medications are taken lifetime.
d. They may adjust their thyroid medication according to the symptoms.
Answer: C – due to inadequate T3 and T4 in hypothyroidism, thyroid preparations are taken lifetime.

92. This is a 45-year-old patient who lost large amount of blood due to gunshot wound (GSW) on the
thigh and abdomen. The patient develops hypotension while receiving transfusion. Which of the
following other symptoms, if seen in this patient, would be most significant in terms of realizing that
a potential complication is occurring?
a. Shortness of breath
b. Back pain
c. Hematuria
d. Proteinuria
Answer: B – back pain in blood transfusion reaction means kidney involvement.

93. Which statement is best to document a patient’s behavior in an unbiased way?


a. “The patient’s hostility created difficulties for the nursing staff.”
b. “the patient’s rudeness prevented administration of his medications.”
c. “The patient threw the water pitcher across the room during shift change.”
d. “The patient’s dressing change was interrupted by his belligerent behavior.”
Answer: C – nursing documentation must be objective.

94. A nurse is administering a shot of Vitamin K to a 3-month-old infant. Which of the following target
areas is the most appropriate?
a. Gluteus maximus
b. Gluteus minimus
c. Vastus lateralis
d. Vastus medialis
Answer: C - Vastus lateralis is the most appropriate location.

95. A patient asks the nurse, “My doctor recommended I increase my intake of folic acid. What type of
foods contain the highest concentration of folic acids”?
a. Green vegetables and liver.
b. Yellow vegetables and red meat.
c. Carrots
d. Milk.
Answer: A – green vegetables and liver are great source of folic acid.

96. A client asks the nurse about her beta-blockers medication effect to her angina. What would be the
nurse’s response?
a. Decrease cardiac output.
b. Increase cardiac output.
c. Decrease cardiac contractility.
d. Increase cardiac contractility.
Answer: C- beta blockers decreases HR and decreases the contraction of the heart muscles.

97. Nitroglycerin SL is prescribed for anginal pain. When teaching how to use nitroglycerin, the nurse
tells the client to place 1 tablet under the tongue when pain occurs and to repeat the dose in 5 minutes
if pain persists. The nurse should also tell the client to place 2 tablets under the tongue when intense
pain occurs.
a. swallow 1 tablet and place under the tongue when pain is intense.
b. place 1 tablet under the tongue 3 minutes before activity and repeat the dose in 5 minutes if pain
occurs.
c. place 1 tablet under the tongue when pain occurs and use an additional tablet after the attack to
prevent recurrence.
Answer: C--anginal pain, which can be anticipated during certain activities, may be prevented by dilating
the coronary arteries immediately before engaging in activity.

98. On admission, the nurse gave the client a dose of epinephrine. The client is having typical side
effects from epinephrine when the nurse observes that he is:
a. Having tremors and bradycardia.
b. Having bradycardia and tachycardia.
c. Having tachycardia and hypertension.
d. Having tremors and hypertension.
Answer: C - Increased BP and Tachycardia are effects of Epinephrine.

99. The patient has been receiving 2500 ml of IV fluid and 300 to 400 ml of oral intake daily for 2 days.
The patient’s urine output has been decreasing and now has been less than 40 ml per hour for the past
3 hours. The nurse should immediately:
a. Catheterize the patient to empty the bladder.
b. Assess breath sounds and obtain the patient’s vital signs.
c. Check for dependent edema and continue to monitor I/O.
d. Decrease the IV flow rate and increase oral fluids to compensate.
Answer: B – the imbalance in intake and output, with a decreasing urinary output, may indicate renal
failure with an increase of body fluid and the beginning development of congestive heart failure; assessing
breath sounds and vital signs are the first steps when monitoring for these complications.

100. When taking the BP of a patient who has AIDS the nurse must:
a. Wear clean gloves.
b. Use barrier techniques.
c. Wear a mask and gown.
d. Wash the hands thoroughly.
Answer: D – blood and body fluid precautions require that hands be washed before and after patient care to
minimize the risk of transmission.

101. Which two percussion sounds are normally heard over the abdomen?
a. Tympany and dullness.
b. Resonance and tympany.
c. Flatness and dullness.
d. Flatness and hyperresonance.
Answer: A –tympany is heard when percussing over hollow organs, such as an empty stomach. Dullness is
heard when percussing over solid organs such as the liver or feces-filled intestines.

102.The nurse is teaching a client and family about phenelzine (Nardil). Which of the following foods
would the nurse urge the client to avoid?
a. Eggs
b. Vegetables
c. American cheese
d. Fruits
Answer: C – cheese is one of the foods that may cause reaction when taking Nardil, a MAOI.
103.When teaching a multigravid client diagnosed with mild preeclampsia about nutritional needs, which
of the following types of diet would the nurse discuss?
a. High-residue diet.
b. Low-sodium diet.
c. Regular diet.
d. High-protein diet.
Answer: D – high protein diet will replace the protein lost in the urine in a PIH patient.

104.Which of the following can be considered balanced diet?


a. Peanut butter sandwich, apple, and milk.
b. French fries, green salad, choco drink.
c. Two pieces of fried chicken, custard cake, chocolate drink.
d. Macaroni and cheese, grilled chicken, green salad.
Answer: A – balance diet consist of CHO, protein and fat.

105.A Registered Nurse is working in a medical unit. Due to an exceptionally busy morning, the nursing
supervisor has decided to send an LPN to help out. As a team leader, which of the following patients
will be assigned to the LPN?
a. A 67 y/o male patient who is going to be discharged to home.
b. A 55 y/o male patient, newly admitted, had chest pain episodes while in the ER.
c. A 70 y/o female, with fractured tibia, on buck’s traction.
d. A 22 y/o female came in with iron deficiency anemia, receiving blood transfusion.
Answer: C – the patient with fractured tibia and on buck’s traction is the most stable and must be assigned
to the LPN.

106.The nurse has performed the initial assessments of 4 patients admitted with an acute episode of
asthma. Which assessment finding would cause the nurse to call the health provider immediately?
a. The patient becomes more “wheezy”.
b. The nurse cannot hear any wheezing in both lungs.
c. The patient’s HR went up to 110 following the administration of albuterol.
d. The patient coughed up large amount of bronchial secretions.
Answer: B - Acute asthma is characterized by expiratory wheezes caused by obstruction of the airways.
Wheezes are a high-pitched musical sound produced by air moving through narrowed airways. Patients
often associate wheezes with the feeling of tightness in the chest. However, sudden cessation of wheezing
is an ominous or bad sign that indicates an emergency -- the small airways are now collapsed.

107.Which of the following statements made by the patient should the nurse include in her documentation
at the end of the shift?
a. ‘That red, small pill is not given to me anymore.’
b. ‘I’ve never seen my attending physician since after surgery.’
c. ‘I did not have any bowel movement for the last 3 days.’
d. ‘It seems that I will never get over with the death of my husband.’
Answer: D – the patient is depressed and has a suicidal tendency.

108.You were about to give AM care, a patient requested for pain medication after returning from surgery
24 hours ago. What do you do first?
a. Take VS, assess pain, do AM care.
b. Give medication, take VS, assess pain, do AM care.
c. Take VS, give medication, do AM care.
d. Perform AM care, give medication, assess pain.
Answer: C – vital signs must be assessed before pain medication is given.

109.Following major abdominal surgery, Mrs. Lafayette, was started on Morphine infusion via PCA
(patient’s controlled analgesia) pump. Priority nursing care for the patient would include:
a. Assess the patient’s respiratory status.
b. Assess the patient’s heart rate.
c. Assess the patient’s level of pain.
d. Check the patient’s abdominal dressing.
Answer: A – morphine sulfate is a CNS depressant.

110.A 6- year- old boy with HIV and fever is being admitted to the unit. The nurse making the room
assignment would place the child in a room with:
a. A 3-year old girl with fractured leg on skeletal traction.
b. A 4-year old boy with croupy cough and a productive cough.
c. An infant girl with a 2-day diarrhea.
d. A 7-year old boy who came in with glomerulonephritis.
Answer: A—this is the safest room assignment for a child with HIV and a fever. A child with HIV should
not be placed in a room with anyone infectious (choices 2 & 3). Due to the fact that he has a fever he
should not be placed in a room with a child who has bacterial infection (glomerulonephritis). The sex
matching is not important unless both children are over the age of 6 years.

111. Which of the following individuals is at greatest risk for experiencing cerebral vascular accident?
a. A 50-year-old African-American male with history of hypertension.
b. A 55-year-old Caucasian female who is obese.
c. A 60-year-old African-American female who smokes.
d. A 45-year-old Caucasian male with hypertension.
Answer: A - This patient has the most factors of CVA: black race, gender, age, and cholesterol.

112.Which of the following female patients is at the highest risk for the development of breast cancer?
a. A 28-year-old who was 10 years old at menarche.
b. A 60-year-old who is multiparous.
c. A 30-year-year-old who is an x-ray technician.
d. A 48 year-old female sexually active.
Answer: A – early menarche is a factor of breast CA.

113.The nurse is caring for a school-aged child with a diagnosis of acute glomerulonephritis secondary to
bacterial tonsillitis. Which of the following lab data should receive priority attention?
a. Markedly elevated BUN and Creatinine levels.
b. Generalized edema.
c. Blood-tinged urine.
d. Significantly poor appetite.
Answer: A – markedly elevated BUN and creatinine levels, in the context of acute glomerulonephritis, may
indicate renal failure or significant fluid deficit.

114.A client has been on antibiotics for 72 hours for cystitis. Which report from the client requires priority
attention by the nurse?
a. Dysuria
b. Body malaise
c. Rashes all over the body and slight breathing problems.
d. Poor appetite.
Answer: C - rashes and breathing problems are signs of delayed allergic reactions to the antibiotics.

115.Thirty-minutes after the blood transfusion was started, the patient complains of urticaria in the chest,
abdomen and both thighs. The PN will implement which nursing action as a priority?
a. Stop the infusion
b. Slow the rate of infusion
c. Take vital signs and observe for further deterioration
d. Administer Benadryl and continue the infusion

116.The patient who was admitted with Emphysema was receiving oxygen at 6L/min via nasal prong. At
the beginning of the shift, during nurses’ rounds, the patient was noted to be lethargic although
arousable; skin is flushed and respirations were down to 8/minute. Priority nursing intervention would
include:
a. Increase the oxygen to 10 liters per minute via facemask.
b. Place the patient in an upright position.
c. Change the oxygen dosage to 2 liter/minute via nasal prong.
d. Call for code blue.

117.Two hours post below knee amputation, the patient’s dressing was saturated with serosanguinous
secretions. In addition to notifying the surgeon, the priority nursing action would include:
a. Monitor vital signs and call MD stat.
b. Apply firm pressure on the stump.
c. Reinforce the dressing.
d. Change the dressing and assess the incision.

118.The nurse is reviewing laboratory results on a client with acute renal failure. Which one of the
following lab values should be reported immediately?
a. Serum sodium 135 mEq/L.
b. Hemoglobin of 9.5 mg/dl.
c. Serum calcium 2.5 mmOs/L.
d. Serum potassium 5.9 mEq/L.

119.On the first following the application of long leg cast, the young girl complains of excruciating pain on
the affected leg. Neurovascular assessment was performed. The PN would report which of the
following findings to the MD?
a. Capillary refill 20 seconds.
b. The child moves her toes but has difficulty raising the leg with cast.
c. Light touch was perceived on the affected leg.
d. The toes were cold to touch; toes wiggled.

120.Marie is a 38-year-old primigravida who is admitted with a diagnosis of pregnancy induced


hypertension (PIH). Priority nursing care on the patient with PIH include:
a. Urinary protein, 2 grams/24hous.
b. HR 110 bpm.
c. Urine output 335 ml/24 hours.
d. Patellar reflex positive.

121.A patient with history of asthma presented himself to the outpatient clinic due to feeling of tightness
on his chest, audible wheezing and mild shortness of breath. Upon arrival at the clinic his peak flow
was at 480 liters/minute but in an hour of arrival, his peak flow has dropped to 200 liters/minute. What
should the nurse do first?
a. Send him ahead to see the physician ahead of the waiting patients.
b. Administer a dose of albuterol inhalation.
c. Administer oxygen at 6 L per nasal prong.
d. Repeat the peak flow reading in 30 minutes

122.An elderly male patient admitted with a diagnosis of R/O cerebral vascular accident has had a
significant change in his blood pressure: BP on admission was 160/100; an hour later BP went up to
180/110. The patient also appears “sleepy”. Which assessment finding should the nurse report
immediately to the provider?
a. Unilateral facial drooping.
b. Tachycardia
c. Muscle weakness
d. Sleepiness

123.An elderly female patient sustained a fractured hip from a fall accident while taking her shower. She
was brought by the EMS to the hospital and was admitted to orthopedic unit. She was placed on buck’s
traction while waiting for hip replacement. Which of the following nursing interventions should
receive priority?
a. Maintaining body alignment.
b. Hourly neurovascular assessments of the affected leg.
c. Turning the patient every 2 hours to prevent skin breakdown.
d. Applying an over-bed trapeze to assist the client with movement in bed

124.On the day of admission of a 55-year-old male patient came in with a diagnosis of acute MI, the
nurse has satisfactorily assisted the patient in prioritizing his concern when she discusses which of the
following with the patient?
a. Back to work guideline.
b. Cardiac rehab guideline.
c. Medication and diet guideline.
d. Activity and rest guidelines.

125.The clinic nurse received a phone from a concern grandmother that her 3 year-old grand child is noted
to be constantly and persistently "scratching his bottom” and disturb sleep at night." Based on this
report, the nurse will initially suspect:
a. Skin disease
b. Scabies
c. Poor hygiene
d. Pinworms

126.Priority nursing diagnosis on the patient who was admitted with acute renal failure would be:
a. “Bed rest to conserve oxygen.”
b. “Increase fluid intake to promote urination.”
c. “Fatigue due to altered nutrition.”
d. “Dehydration secondary to diuresis.”

127.Leno, 35- year-old of African descent, came to USA to visit his family. He was admitted in the
hospital due to exacerbation of sickle cell. During the vaso-occlusive crisis, which of the following
nursing interventions is a priority?
a. IV at 150 ml/hour; analgesia; oxygen at 10L via facemask.
b. Bedrest; hourly vital signs; analgesia.
c. Warm compresses to painful sites; limit fluids; bedrest.
d. Bedrest and apply splints to painful joints; analgesia; bedrest.

128.At the well-baby clinic, the nurse follows certain sequence of the baby’s assessment that includes:
a. Weight; TPR.
b. RR, HR. weight
c. TPR; weight
d. Temp; weight; RR; HR

129.On the second day of surgery, the mother reported fever and rapid respirations on her child. The nurse
check the child’s temperature and it was 39°C. The first action taken by the nurse should be to
a. inform the physician ASAP.
b. encourage the child to drink more fluids.
c. give a tepid sponge bath.
d. administer the prescribed acetaminophen

130.While retrieving her voice messages, the public health nurse received a call from the patient who is
receiving short acting insulin for her diabetes. The patient states, “I know I have a severely low sugar
since the regular insulin was given 3 hours ago and it peaks in 2 hours.” When the nurse returns the
call, the first question she will ask the patient would be?
a. “Do you know how to give insulin shots?”
b. “Have you eaten since you’ve received the insulin?”
c. “What are you doing at this moment?”
d. “Will you be able to see your physician tomorrow?”

131.The nurse would question which of the following orders prescribed on the patient with ketocidosis?
a. Run IV NS at 100 ml/hour.
b. Perform acu checks for blood sugar every 2 hours.
c. Administer NPH 20 units SC.
d. Monitor hourly urine output.

132.Gastrostomy tube was placed on the patient with terminal stage of throat cancer. Feeding is started
today. Prior to initiating the first gastrostomy feeding, the nurse should perform which of the
following?
a. Measure residual feeding.
b. Palpate the abdomen.
c. Assess for breath sounds.
d. Flush the tube with 50 ml of NS.

133.A child with a croup has difficulty of breathing. One of the following interventions, if performed by
the nurse should be a priority and most important for the patient:
a. Administer oxygen.
b. Suction the client.
c. Place the client in high fowler’s position.
d. The client should stay in side lying position.

134.At term, Ms. Star is admitted to the hospital in active labor which progresses normally. She delivers a
baby boy. Ms. Star is transferred to the postpartum unit. A few hours after Ms. Star’s delivery, the
nurse notes that Ms. Star has saturated two perineal pads with blood within a 20-minute period. Which
of the following actions should the nurse takes first?
a. Check the consistency of the client’s uterine fundus.
b. Encourage the client to void.
c. Take the client’s BP.
d. Notify the nurse in charge.

135.Post thoracentesis, the patient complains of blood-tinged sputum. Which of the following would
the PN performs first?
a. Apply pressure on the site.
b. Auscultate the lungs for absence of breath sounds.
c. Obtain sputum for C & S.
d. Inform MD.

136.Which of the following patients should you take care of first?


a. A patient who is incontinent and needs to be changed.
b. A patient who needs to be suctioned.
c. A patient who needs her dressing changed.
d. A patient who needs pain medication.

137.Which of the following nursing diagnoses would receive the greatest priority in the care of an
unconscious patient with a head injury?
a. Impaired gas exchange related to shallow irregular breathing
b. Risk for injury related to disorientation and decreased level of consciousness
c. Disturbed Sensory Perception related to decreased level of consciousness
d. Ineffective Airway Clearance related to inability to remove respiratory secretions.

138.When making rounds on assigned patients, which of the following patients would the nurse assess
first?
a. A 16-month-old with cellulitis on the L leg who is to be discharged today.
b. A 7-year-old who had an appendectomy yesterday and complains of postop pain.
c. A 10-year-old who has just been admitted in sickle cell crisis.
d. A 16-year-old receiving a third day of chemotherapy.

139.Two days postpartum a patient who had a C-section delivery complains of pain in the right leg. The
nurse should instruct the patient to
a. apply warm compresses.
b. massage the affected area.
c. encourage range of motion exercises to promote circulation.
d. maintain bed rest and notifies the physician.

140.Stephen is a 58-year-old male patient who has a long history of alcoholism. He develops Laennec’s
cirrhosis, has ascites, jaundice and is confused. The nursing priority for Stephen is to
a. institute safety measures.
b. place him in an upright position.
c. measure the abdominal girth every day.
d. test each stool for the presence of blood.

141.A woman with an internal fetal monitor experiences a variable deceleration. Your priority nursing
intervention would be:
a. Administer oxygen
b. Administer Pitocin.
c. Administer Demerol.
d. Monitor the mother’s vital signs

142.ThE PN may appropriately delegate which of the following to the nursing assistant?
a. Take the patient’s vital signs before starting a blood transfusion.
b. Change the patient’s central line dressing.
c. Take blood pressure on a patient complaining of chest pain.
d. Suction the patient’s tracheostomy tube.

143.Which of the following nursing interventions is a priority on a newborn baby?


a. Wrap the baby in a warm blanket.
b. Place the baby in a side lying position.
c. Appraise the baby using the “Apgar Scoring System”.
d. Administer Vitamin K IM.

144.A pregnant patient came in and presents with prolapsed cord. What will be your initial action?
a. Give oxygen 4-5 L/min.
b. Cover the cord with sterile towel.
c. Let the patient assume knee chest position.
d. Notify the doctor.

145.A 69-year-old woman has a history of heart failure. She is admitted to the emergency department
with heart failure complicated by pulmonary edema. On admission of this patient, which of the
following should be assessed first?
a. blood pressure
b. skin breakdown
c. serum potassium
d. urine output

146.When providing home care to a group of clients, which of the following would require immediate
referral to the MD?
a. A 65-year-old prostate cancer.
b. A 47- year- old with Meniere’s disease complaining of feeling dizzy.
c. A 72 –year- old with Alzheimer’s disease complaining of disliking his home attendant.
d. A 32 –year- old with history of fall complaining of vomiting upon waking in the morning.

147.A nurse assigned to the pediatric unit has the following patients. Which one should be given priority?
a. A 2-month-old with a sunken anterior fontanel.
b. A 4-month-old with altered consciousness.
c. A 6-month-old crying inconsolably.
d. A 7-month-old who cannot hold his own bottle.

148.The nurse assesses a patient’s IV site and finds that it is red, hot, and tender to touch. The most
appropriate first nursing action is:
a. Stop the infusion and remove the IV.
b. Apply a warm soak to the site and reevaluate.
c. Stop the infusion and restart the IV in the same area.
d. Call the physician to evaluate.

149.The nursing intervention that should be instituted immediately to relieve the symptoms associated
with the patient’s hypoglycemic reaction include:
a. Giving 8 oz of fruit juice with 2 tablespoon of sugar.
b. Administering 5% dextrose solution IV.
c. Withholding a subsequent dose of insulin.
d. Providing a snack of cheese and dry crackers.

150.A patient has a bone marrow aspiration performed. Immediately after the procedure, the nurse should:
a. Position the patient on the affected side.
b. Begin frequent monitoring of vital signs.
c. Cleanse the site with an antiseptic solution.
d. Briefly apply pressure over the aspiration site.

151.The nurse is assigned to a patient newly diagnosed with active tuberculosis. Which of these
interventions would be a priority for the nurse to implement?
a. Have the client cough into a tissue and dispose in a separate bag
b. Instruct the client to cover the mouth with a tissue when coughing
c. Reinforce that everyone should wash their hands before and after entering the room
d. Place client in a negative pressure private room and have all who enter the room use masks with
shields

152. After paracentesis, the nurse is doing the best action when she
a. takes patient’s BP.
b. observes the patient’s abdomen.
c. palpates the patient’s pulses.
d. monitors the patient’s VS.

153. A 2-gram sodium diet is prescribed for a patient with severe hypertension. The
patient does not like the diet, and the nurse hears the patient tell a friend to bring
in some ‘good home-cooked food.’ It would be most effective for the nurse to plan
to
a. call the dietician for patient teaching.
b. wait for the patient’s family and discuss the diet with the patient and family.
c. tell the patient that the use of salt is forbidden.
d. catch the family members before they go into the patient’s room and tell
them about the diet.

154. Three days following surgery, a client with cast on his right leg would like to
ambulate for the first time. He has not been seen by the physical therapist. For
the last three days he has been sitting on the edge of the bed with feet dangling.
Today, he called the nurse to walk him to the washroom. Using a sound judgment,
the nurse’s most appropriate response would be which of the following?
a. Assist the client to the bedside commode.
b. Do not allow the client to ambulate, tell him to use the bedpan.
c. Ask another nurse to help as you walk him to the washroom.
d. Consult the physician about the client’s activity.

155. The family of a terminally-ill client asks the nurse to help them end the
suffering of her loved ones. Which of the following would be the most appropriate
response by the nurse?
a. “Euthenasia is under the criminal code.”
b. “This is against the code of nursing ethics.”
c. “You should not lose hope as long as your loved ones lives.”
d. “This is against the standard of nursing practice.”

156. The doctor ordered an important drug for you’re your client, which should be
administered on time. You will take one of the following actions because you have
not received the drug on time:
a. Try to explain to the client that you have not received the drug from the
pharmacy.
b. Call the pharmacy and ask them to deliver the drug ASAP and inform the
physician.
c. Inform the physician about the delay of the drug and ask him to re-order the
drug.
d. Inform your nursing supervisor.

157. A client who had previously signed the consent for liver biopsy has changed
his mind and no longer wants the procedure. The best initial response by the
nurse would be:
a. “Why did you originally sign the consent?”
b. “Can you tell me why you decided to refuse the procedure?”
c. “You are obviously afraid about something concerning the procedure.”
d. “Although the procedure is very important, I understand why you changed
your mind.”

158. A 6- year- old boy with HIV and fever is being admitted to the unit. The nurse
making the room
assignment would place the child in a room with:
a. A 3-year old girl S/P (status post) appendectomy.
b. A 6-year old boy with wheezing and a productive cough.
c. An infant girl with a rash.
d. A 7-year old boy who just finished the induction phase of chemotherapy.

159. The nurse is conducting an intake interview with a new patient. Which of the
following statements by
the nurse reflects assessment on spiritual beliefs?
a. “How would you describe your faith?’
b. “Do you belong to a specific religion?”
c. “Do you wish to see the hospital clergy?”
d. “What is your religion?”

160. Liang Tam , 68 years old, was admitted to the hospital with GI bleeding. She
arrived to USA from
China, as landed immigrant, 3 months ago. Chin speaks Mandarin and little
English. Her daughter,
Linda Min, speaks fluent English. To whom will the registered nurse, during the
admission interview,
collects the information from?
a. Daughter and patient.
b. Interpreter.
c. Ask for previous medical records.
d. Entire family.

161. You were walking in the hallway of the long term facility, you found Jeffrey
surrounded by friends and
visitors. He was unresponsive, no pulse, and no breathing. Some people around
him were screaming for
help and others were blaming each other for what had happened to Jeffrey.
Which one is the most
appropriate action taken by the nurse?
a. Initiate CPR.
b. Call for help.
c. Clear the hallway with visitors.
d. Dial 911.
162. Rob, 23 years old male patient, admitted to the surgical unit following a
gunshot wound to his L chest
& R thigh. He had a GCS of 15 at the scene. He has a history of ETOH abuse and
a known cocaine
user. During the patients’ round, the nurse has witnessed a visitor handling a
plastic bag with white
powder and syringe to the patient. Which action taken by the nurse is most
appropriate?
a. Call the security guard.
b. Confiscate the stuff.
c. Inform the nursing supervisor.
d. Tell the visitors to get his stuff & leave the room.

163. The patient is admitted to the emergency department and was brought to the
unit due to poor diabetic
management. The wife asks how they can learn better. They have previously
been to a diabetic class
without much success. Which of the following nursing strategy will be
appropriate for the client?
a. Arrange a meeting with the health care team.
b. Have a one-on-one diabetic teaching with the client.
c. Refer the client and wife to group diabetic teaching class.
d. Discuss the possibility of having to stay longer in the unit to manage the
blood sugar.

164. Nurse Mary is scheduled to attend an in-service program but she still has to
perform some nursing care
on her assigned client. Which action will best meet her educational needs?
a. Nurse Rose should be asked to complete the procedure so that Nurse Mary
can attend.
b. Nurse Mary should complete the procedure then attend.
c. Nurse Mary should ask for notes from those who have attended.
d. Nurse Mary should attend on her day off.
165. A colleague came in to work in your unit with scent of alcohol in his uniform.
When asked, he claimed
that on his way to work, a gentleman accidentally spilled a glass of wine on his
shirt. However, you noted that his breath smells alcohol. Which of the following
nursing actions will be best for the nurse to take?
a. Send your colleague home.
b. Inform the nursing supervisor.
c. Assign your colleague to semi-conscious patients.
d. Ask your colleague to do deskwork.

166. A 14-year-old patient with a diagnosis of gonorrhea requested you not to tell
her parents about the
disease. How will you respond?
a. “Sorry but your parents must be informed.”
b. “It will remain confidential as requested.”
c. “I have nothing to do with this. This is your private life.”
d. “Since this concern your diagnosis, I need to inform your parents and your
school principal.”

167. A nurse is going to give health teachings to a group of 11 to 15 years old


clients with Diabetes Mellitus.
Considering the group’s developmental needs, the type of learning activity
planned by the nurse would
be:
a. Include the parents in the teaching sessions.
b. Teach about physiology of puberty.
c. Ensure participants make independent decisions about their treatment.
d. Plan activities that will encourage peer-support.

168. Which of the following makes the final decision in revoking a Registered
Nurse’s license registration?
a. Registered Nurse of New York
b. Nurses ASSOCIATION
c. Registered Nurses Association.
d. New York State Nurses Association

169. The nurse on the floor will perform peripheral IV site insertion. Her
performance is based on:
a. Hospital policies and procedures.
b. Nursing Standards of Practice.
c. Doctor’s orders.
d. IV regulation developed by the Board of Nursing

170. A patient walked in an extremely busy emergency room. While waiting for his
turn to been seen by the
nurse, he suddenly became very aggressive and was demanding to see a doctor.
He came forward to the nurse in a threatening stance. Which of the following
actions made by the nurse promotes her safety?
a. The nurse informs the patient to wait for the doctor.
b. The nurse moves backward to create a safe distance from the patient.
c. The nurse calls the security guard.
d. The nurse ignores the patient.

171. What should the nurse do in order for the participants get necessary
information about changing their
attitudes towards the issues of breast cancer prevention?
a. Invite a speaker from the Cancer Society.
b. Invite a breast cancer survivor.
c. Distribute pamphlets regarding the cause and prevention of breast cancer.
d. Distribute questionnaires in order to determine the number of interested
people in the
community.

172. The nurse documents and performs which of the following when the patient
refuses his medication?
a. Kardex; physician.
b. Medication record; physician.
c. Medication record, progress notes; informs the physician.
d. Incident report form; physician.

173. After the patient has refused the medication, the next step taken by the nurse
will be:
a. Document the patient’s refusal.
b. Explore why the patient refuses.
c. Inform the nurse-in-charge about the refusal.
d. Notify the physician immediately.

174. The nurse who has explored the patient reason for refusing the medication will
a. Document the patient’s refusal.
b. Inform the nurse-in-charge about the refusal.
c. Notify the physician.
d. Inform the patient’s family.

175. The mother’s condition has deteriorated during labor and a cesarean section
is required. Her husband
is worried and anxious about the changed of plan and demand for an
explanation. Who among the
following should explain the procedure to the husband?
a. The surgeon.
b. The attending physician.
c. The primary nurse.
d. The medical team.

176. A 13-year-old female patient went to the clinic and asked for contraceptive
pills. Following her visit the
nurse received a phone inquiry from the patient’s mother regarding the visit. The
nurse’s most appropriate statement is:
a. “I refuse to answer your question.”
b. “Your daughter came for some contraceptive pills.”
c. “I am too busy right now. Please call me back later.”
d. “It is not appropriate to disclose this information.”

177. Lynn Tyrell 60 years old is attending the group therapy when all of a sudden
she collapses and the
nurse
a. initiates CPR.
b. performs chest compression.
c. checks the patient for unresponsiveness.
d. calls for code blue.
178. When giving NG feeding, the nurse was observed of not checking the NGT
placement prior to
administering the feeding in spite of previous instructions from a colleague.
Appropriate action taken
at this time would include:
a. Report the incident to the nursing supervisor.
b. Confront your colleague.
c. Remind your colleague once again.
d. Fill up an incident report.

179. A male client in the dermatologist’s clinic asked the nurse for information on
contraceptives. The
nurse, who was aware of her limited knowledge on contraceptives, has
provided contact number for
information to the client. The nurse’s action was based on the Regulatory
Board’s standard of practice
which is:
a. Accountability
b. Fairness
c. Competency
d. Sympathy

180. The typical acute asthmatic attack is usually due to a combination of bronchial
spasm, bronchial edema, and the presence of mucoid secretions in the bronchial
lumen. The client with asthma was given mucomyst for which of the following
reasons?
a. To decrease the viscosity of secretions.
b. To encourage the movement of the secretions in the bronchi.
c. To stimulate the cough reflex.
d. To dilate the bronchi.

181. Most asthmatic attacks respond to Epinephrine (Adrenalin) and it may be used
as a therapeutic agent in the treatment of some allergic conditions because it:
a. Improves cardiac output due to its positive inotropic effect.
b. Acts as a hypertensive agent elevating BP.
c. Dilates the bronchi.
d. Antagonizes histamine.

182. Twenty-four hours following burn accident, Johnny was given Morphine Sulfate
to control his pain.
Which of the following parameters should the nurse utilize to accurately
measure the client’s
response to drug therapy?
a. How the client feels.
b. Use the pain scale system.
c. Pay attention to non-verbal cues.
d. Monitor the client’s vital signs.

183. To assess the effectiveness of a vasodilator administered to lower


hypertension, the nurse should take the client’s pulse and blood pressure:
a. Prior to administering the drug.
b. Thirty minutes after giving the drug.
c. Immediately after the patient gets out of bed.
d. After the patient has been lying flat for 5 minutes.
184. Mr. Lou, 65 years old, with liver cirrhosis was ordered neomycin enema for
which of the following goals?
a. To lower the client’s high BP.
b. To improve the client’s level of consciousness.
c. To promote diarrhea.
d. To correct constipation.
185. A male, Hispanic client with history of excessive ‘alcohol-drinking’, was
admitted with a diagnosis of recurrent pancreatitis. On admission, the client was
complaining of unbearable pain. The nurse knows that the appropriate narcotic
analgesic for this client would be
a. Demerol.
b. Morphine.
c. Tylenol #3.
d. Ativan.

186. Following Lasix therapy on a client with heart failure, the nurse would assess
which of the following reflecting the desired effect of the drug:
a. Electrolyte imbalance.
b. Vital signs.
c. Diuresis.
d. Hypotension.

187. Following Nitroglycerin patch application, an expected outcome experienced


by the client would be one of the following:
a. Elevated blood pressure
b. Tachycardia
c. Tingling sensation
d. Increased urine output

188. When teaching a client about antacid therapy, the nurse should include the
fact that antacid tablets:
a. Are taken 1 hour before meals.
b. Are as effective as the liquid forms.
c. Should be taken only at 4-hour intervals.
d. Interfere with the absorption of other drugs.

189. A client with tuberculosis has been receiving INH as part of the
chemotherapeutic therapy protocol. During a subsequent clinic visit the nurse
should recognize that prompt intervention is required when the client develops
a. yellow sclera.
b. orange stools.
c. a temperature of 98.6˚F.
d. a weight gain of 5 lb.

190. A nurse picks up an order for an asthmatic child. The order reads: Propranolol
(Inderal) 80 mg. PO QD. The nurse reads this order as
a. Appropriate dose.
b. Contraindicated.
c. Inappropriate dose.
d. Indicated.

191. A manic client is being treated with lithium carbonate. The nurse would
correctly suggest which of the following activities for this client.
a. Playing solitaire.
b. Basketball
c. Distribution towels to other patients’ rooms.
d. Monopoly

192. The nurse knows that the maximum amount of fluid that can safely be
injected intramuscularly in an infant is
a. 0.5 cc.
b. 1.0 cc.
c. 1.5 cc.
d. 2.0 cc.
193. Which of the following signs should the nurse be aware of on a patient who is
taking prednisone?
a. Melena
b. Hypoglycemia
c. Hypekalemia
d. Low platelet counts

194. Ms. Cox is an adult ♀ patient who was prescribed depo-provera. The drug was
prescribed for which of the following?
a. It is a contraceptive drug given by IM injection every 3 months.
b. It is a type of hormonal replacement therapy given every 3 months by IM
injection.
c. It is a drug that prevents osteoporosis.
d. It is the drug of choice for those who are planning to get pregnant.

195. Mary Jane is 7-year-old girl who was admitted with asthma exacerbation.
Admission vital signs include a HR of 120 bpm, RR 34, breathing was wheezy on
ausculatation, Temp 99.5˚F. Based on the admission assessment, the nurse will
expect which drug will be ordered on the patient ASAP?
a. Intal
b. Ventolin
c. Mucolytics
d. Solumedrol IV

196. Mrs. Byron rushes into the ER carrying 2-year-old Amanda in her arms. She
thrusts a half-empty bottle of aspirin at the nurse and states that he saw Amanda
eating them just 30 minutes earlier. After confirming that Amanda had eaten at
least six tablets of aspirin, the nurse should
a. Give her 15 milliliters of ipecac syrup and a large glass of water
b. Prepare her for gastric lavage
c. Catheterize her for a fresh urine sample
d. Start an intravenous infusion for administration of an antidote

197. Danielle Knetchel, 32 years old, experiences diarrhea after eating in the
restaurant. She went to the
clinic and the physician prescribed anti-diarrheal drug, a narcotic that causes
dryness of the mouth.
Which of the following drugs was given to Danielle?
a. Gravol
b. Immodium
c. Chew lemon
d. Lomotil

198. The physician ordered Lomotil 5 mg p.o. QID on Mr. Spikes for his diarrhea.
Expected side effect of the drug would include:
a. Urinary retention.
b. Decreased peristalsis.
c. Tinnitus
d. More diarrhea

199. The nurse administers prescribed pancreatin replacement therapy to Bonnie.


To effectively evaluate
the effect of this treatment, should expect that this medication will result to
a. Bulky, foul, smelly stools.
b. Close to normal stools.
c. Loose, frequent stools.
d. Constipation.

200.Mr. Holland is a 55-year-old male patient who has a long history of ETOH abuse. He presently
undergoes evaluation tests for suspected liver cirrhosis. The nurse understands that a liver biopsy may
be contraindicated in certain situations. Therefore, it is important for the nurse to assess the patient for:
a. confusion and disorientation
b. the presence of any infectious disease
c. a prothrombin time of less than 40% of normal
d. foods high in Vit K eaten before the biopsy

201.The nurse understands that Mr. Holland’s ascites related to cirrhosis is most likely the result of:
a. Impaired portal venous return
b. Inadequate secretion of bile salts
c. Excess production of serum albumin
d. Decreased interstitial osmotic pressure

202.One of the following is not a symptom of the patient with right sided heart failure:
a. Weight gain
b. Elevated CVP reading
c. Flattened neck veins
d. Abdominal discomfort

203.A client’s monitor shows a PQRST wave for each beat and indicates a rate of 120. The rhythm is
regular. The nurse should note that the client is experiencing:
a. atrial fibrillation
b. sinus tachycardia
c. ventricular fibrillation
d. first-degree atrioventricular block

204.Susan sustains a needle stick injury while giving IM injection on a patient who is diagnosed with
AIDS. Which of the following is the most important action that Susan should take?
a. Report the incidence to the occupational health.
b. Start prophylactic AZT treatment.
c. Wash the affected site with soap and warm water.
d. Go to the ER for treatment.

205.A new mother has some questions about PKU. Which of the following statements made by the nurse is
correct regarding PKU?
a. An amniocentesis is required prior to delivery.
b. PKU test is performed soon after birth.
c. PKU is preventable if the mother eats adequate amount of protein during pregnancy.
d. The effects of PKU are irreversible

206.A patient who has recently experienced a thrombotic CVA will most likely receive which of the
following treatments?
a. Streptokinase
b. Morphine
c. Heparin
d. Coumadin

207.The mother of the child with Leukemia asks the nurse about the life span of the RBC. The correct
response is:
a. RBC lives for few hours.
b. RBC stays in the body for about 3 days.
c. RBC lives as much as WBC lives.
d. RBC’s life span is 120 days.

208.Binu is a 45-year-old female patient who is admitted with a diagnosis of acute depression. She has
history of inability to deal with her divorce. The nurse would expect that one of the following drugs
will be ordered for the patient which is:
a. Elavil
b. Ativan
c. Valium
d. Tylenol #3.

209.If Larissa, an IDDM, is in acute ketoacidosis, the nurse would expect to see which of the following
group of symptoms?
a. Vomiting; extreme thirst; diuresis; coma.
b. Extreme thirst; cold clammy skin; weakness.
c. Weight loss; weakness; cold clammy skin; tremors.
d. Acetone breath smell; lip tremors; weakness.

210.Which of the following assessment data is present in all kinds of shock?


a. Decreased peripheral vascular resistance.
b. Tachycardia; hypervolemia.
c. Hypovolemic; tachycardia.
d. Hypotension; increased cardiac output.

211.Anaphylactic shock is a serious type of blood transfusion reaction. The pathophysiology of


anaphylaxis is
a. It is treated with adrenalin or epinephrine.
b. A severe vasoconstriction and profound bronchodilation.
c. Characterized by presence of rashes all throughout the body.
d. A profound hypotension due to profound vasodilation and severe bronchoconstriction.

212.The patient in anaphylactic shock is treated with emergency drugs. These drugs aim to reverse the
pathophysiologic changes developed in anaphylactic shock. Which medications are usually given?
a. Epinephrine and aminophylline.
b. Epinephrine and adrenaline.
c. Epinephrine and Benadryl.
d. Amiophylline and Benadyl.

213.Six hours following bilateral hip replacement, an 80-year-old woman, transferred in from the Recovery
Room to the Surgical Unit was observed to be confused and lethargic; VS are BP 82/56; HR 120; RR
28; skin cool and clammy; no c/o SOB; lungs sounds clear; u/o 20 ml/hour; T 36.5°C; WBC 9000.
Based on this information, what condition appears to be developing?
a. Neurogenic shock.
b. Distribute shock.
c. Septic shock
d. Hypovolemic shock

214.A patient with complaints of loss of peripheral vision is suspected to have


a. cataract.
b. retinal detachment.
c. glaucoma.
d. myopia.

215.A mother with cystic fibrosis child is receiving health teaching from the nurse regarding danger signs
of her illness. The mother has best understood the teaching when she says:
a. “I will be alarmed when child has fatty stools.”
b. “I will have to report immediately when my child has salty taste.”
c. “I will have to monitor my child respirations when he gets tired.”
d. “I will take my child to the doctor once if he is coughing.”

216.A patient came to an emergency room and complaints that he is having pain in the right lower
abdominal quadrant and fever 2 days duration. He said to the nurse that he might be having
appendicitis. Which of the following information may confirm the patient’s complaint?
a. Positive rebound tenderness on physical assessment.
b. Patient’s abdominal pain is more severe when abdominal pressure is applied.
c. The client’s temperature is 39°C.
d. The patient’s WBC is 12,000.

217.The patient is taking antidepressant drug, Elavil. The spouse of the patient complains that the dosage of
the drug needs to be increased because no improvement is noted. The patient has been taking the drug
for the last 3 days and no improvement is noted. The most appropriate nursing response would be:
a. “I will inform the physician about your concern.”
b. “It may take 3 to 4 weeks for the drug to work.”
c. “The drug should work right away.”
d. “Possible drug resistant has developed.”

218.On the patient with pancreatitis, which of the following is decreased in level?
a. Urine amylase
b. WBC
c. ESR
d. RBC

219.Which of the following is the function of the frontal lobe?


a. Recognized between right and left.
b. Responsible for critical thinking.
c. Recognize colors.
d. Interpret body needs.

220.The child with a diagnosis of laryngotracheobronchtis should be observed for:


a. Expiratory wheezing.
b. Inspiratory stridor and lethargy.
c. Deep, rapid breathing.
d. Shortness of breath.

221.The client is taking Aminophylline. The nurse knows that the client is having problem if she observes
that the client is:
a. coughing
b. restless
c. teary eyes
d. bradycardia

222.The nurse is right when she explains to the client on how to take Dexamethasone:
a. Take it with food.
b. Take it without food.
c. Take it at bedtime.
d. Take it with other meds.

223.The patient with hypothyroidism will most likely experience which of the following?
a. Diarrhea and weight gain.
b. Warm skin and brittle nails.
c. Dry skin and weight gain.
d. Constipation and tachycardia.

224.Following a successful treatment from hypothyroidism, the patient is discharged with Synthroid.
Which of the following discharged instruction is inappropriate?
a. The drug is taken lifetime.
b. Heart rate must be assessed prior to taking the drug.
c. The physician must be informed if the HR is above 100/minute.
d. Take the drug must be taken before bedtime.

225.Which of the following group of clinical manifestations are seen hyperthydoism?


a. Palpations, tachycardia, cold intolerance.
b. Weight gain, irritability, heat intolerance.
c. Tachycardia, weight loss, diarrhea.
d. Weigh gain, large amount of food intake, heat intolerance.

226.The nurse anticipates that one of the following medications will be given to the patient with
hypothyroidism:
a. IV calcium
b. Iodides
c. Prophylthioracil
d. Synthroid

227.Which of the following is the most important physical assessment parameter the nurse would
consider when assessing fluid and electrolyte balance?
a. Skin turgor
b. Intake and output
c. Osmotic pressure
d. Cardiac rate and rhythm

228.When assessing a patient for signs of fluid overload, the nurse would expect to observe
a. bounding pulse.
b. flat neck veins.
c. poor skin turgor.
d. vesicular breath sound.

229.The physician has ordered IV replacement of potassium for a patient with severe hypokalemia. The
nurse would administer the IV potassium
a. by rapid bolus.
b. diluted in 100 cc over I hour.
c. diluted in 10 cc over 10 minutes.
d. IV push.

230.When monitoring a patient who is receiving a blood transfusion, the nurse would analyze an elevated
body temperature as indicating
a. a normal physiologic process.
b. evidence of sepsis.
c. a possible transfusion reaction.
d. an expected response to the transfusion reaction.
231.Mrs. Jones is receiving Digoxin and Lasix daily. Today, Mrs. Jones complains of nausea and her apical
pulse is 130 and irregular. Which of the following nursing interventions is the most appropriate?
a. Hold Digoxin and check the patient’s potassium level.
b. Remove the orange juice from the patient’s tray.
c. Identify the patient as high risk for hyperkalemia.
d. Assess the patient for other signs of hypernatremia.

232.Which of the following clinical conditions exacerbates electrolyte excretion?


a. Nasogastric feedings
b. Use of surgical drains
c. Immobility from fractures
d. Chronic water drinking

233.Disease of which of the following structures is most likely to affect electrolyte re-absorption?
a. Glomerulus
b. Renal tubules
c. Bladder
d. Renal pelvis

234.A diet containing the minimum daily sodium requirement for an adult would be
a. a no-salt added diet.
b. a 2 gram sodium diet.
c. a 4 gram sodium diet.
d. a 1500 calorie weight-loss diet

235.A patient with a diagnosis of bipolar disorder has been drinking copious amounts of water and voiding
frequently. The patient is experiencing muscle cramps, twitching, and is reporting dizziness. The
nurse checks lab work for
a. complete blood count results, particularly the platelets.
b. electrolytes, particularly the serum sodium.
c. urine analysis, particularly for the presence of white blood cells.
d. EEG.

236.The nurse assesses a patient’s IV site and finds that it is red, hot, and tender to touch. The most
appropriate first action is:
a. Stop the infusion and remove the IV.
b. Apply a warm soak to the site and reevaluate.
c. Stop the infusion and restart the IV in the same area.
d. Call the physician to evaluate.

237.A patient is admitted to the hospital with leukemia. The doctor ordered chemotherapy twice a month.
How do you prevent the side effects of chemotherapy?
a. Increase oral fluid intake.
b. Give laxative/ stool softener.
c. Give foods high in protein.
d. Advice the patient to lie down after taking the medicine.

238.The patient delivered vaginally in small community. The nurse is assessing a term neonate about
neonate rooting reflex by.
a. Stroking the neonate’s cheeks.
b. Stroking the neonate’s nose.
c. Stroking the neonate’s chin.
d. Stroking the neonate’s back.

239.A 30-year-old pregnant mother, in her third trimester, asks the community nurse what position is best
for her to prevent heartburn. The nurse correctly recommends the
a. lying position after meals.
b. low fowler position, 15 – 30 degrees angle.
c. high fowlers position to promote ventilation.
d. ensure the HOB is elevated correctly (4 – 6) blocks and not down during the 1st 2 – 3 hours after
food.

240.A patient is admitted to the hospital with the diagnosis of autoimmune compromise syndrome, has an
order of Prednisone, tablet 5 mg PO BID. She is on strict isolation room and visitor has to be
monitored. What is your nursing action in giving the medication?
a. Give the medication at 0800 with food then 2000 with milk.
b. Check the ID band of the client.
c. Document at MAR sheet.
d. Ask the client name.

241.The incidence of spinal cord defects/ neural tube defects such as meningomyelocele has decreased
because
a. prenatal diagnosis by ultrasound and treatment in the uterus.
b. maternal ingestion of folic acid prior and during pregnancy.
c. decreased incidence of smoking and alcohol use during pregnancy.
d. genetic testing prior to pregnancy.

242.What is elevated in the laboratory result to a patient with cystitis?


a. Hemoglobin
b. Leukocyte casts
c. Erythrocyte level
d. Albumin

243.A patient with cystic fibrosis has an order of aerosol treatment and physiotherapy. How will you carry
out this order?
a. Physiotherapy then aerosol.
b. Aerosol then physiotherapy.
c. Do it at the same time.
d. Do it according to nurse convenience.

244.The school nurse found out that the boy in the class has head lice. What is your nursing priority?
a. Shampoo the boy as soon as possible.
b. Notify the public health nurse.
c. Inform the mother.
d. Shampoo all the students.

245.The patient is receiving Lasix. The doctor made his round this morning and ordered ampicillin
medication. The patient is to receive 3 grams of ampicillin per day. The ampicillin available stock is
500 mg/capsule. How many capsules would you administer to the patient?
a. 500 mg- 6 capsules.
b. 250 mg – 6 capsules.
c. 250 mg – 4 capsules.
d. 500 mg – 4 capsules

246.Which of the following nursing assessment is appropriate on a patient who came in with a diagnosis of
appendicitis?
a. Deep palpation
b. Light palpation
c. Auscultation
d. Percussion

247.Classic symptoms of appendicitis:


a. Diffused abdominal pain occurs before nausea, fever.
b. Pain in the right side of the abdomen, fever, elevated WBC.
c. Pain and decreased sensation in the right leg.
d. Distended abdomen, indigestion-like symptom.

248.The patient with DM is for discharged. The nurse who makes home care plan will take which one as
priority?
a. Glucose monitoring.
b. Signs of hypoglycemia.
c. Ketones monitoring.
d. Foot care.

249.The patient with a diagnosis of DM is overweight. Which one of the following food selection is
appropriate?
a. Skim milk
b. Fresh fruits
c. Fresh vegetables
d. Bread

250.Which of the following activities will be appropriate on the patient who was 7- days post above-the-
knee amputation?
a. Muscle exercises
b. Ambulate with walker
c. Bed rest
d. Breathing exercises

251.Seven days post above-the-knee amputation (AKA), recommended activity for the patient would
include:
a. Sit up on bed.
b. Ambulate with walker.
c. Sit up on chair.
d. Ambulate with crutches.

252.The mother brought her child to the clinic. Physical assessment reveals that the child is smaller and
underweight for his age. Which normal findings complement the physical assessment?
a. The boy is Asian
b. The boy is from Europe.
c. The boy is American.
d. The boy is probably malnourished.

253.The patient with DM has flu. Which nursing action is more appropriate?
a. Frequent monitoring of blood glucose.
b. Expected increase in the patient insulin requirement.
c. Implement respiratory isolation.
d. Monitor the patient’s respiratory status frequently.

254.The mother brought her toddler for immunization. Before administering the drug, which important
question should be asked by the nurse to the mother?
a. History of allergy
b. Any reactions to the vaccine.
c. Last immunization.
d. Preferred site of immunization.

255.A teen-ager was diagnosed with iron-deficiency anemia. Appropriate diagnostic test would be:
a. Hematocrit
b. Hemoglobin
c. WBC
d. CBC

256.Michael has congenital Hirschprung disease. Chronic constipation has been a major problem with the
patient. Important finding would include
a. Hard stools.
b. Well formed stools.
c. Ribbon-like stools
d. Large, bulky, foul-smelly stools.

257.Which of the following is the appropriate narcotic-analgesic ordered to provide pain relief on the
patient with pancreatitis?
a. Codeine 30 mgs.
b. Dilaudid 60 mgs.
c. Demerol 75 mgs.
d. Tylenol #3, 2 tabs.

258.In addition to weakness and fatigue observed on the patient with severe anemia, another manifestation
(s) include
a. Cold, clammy skin.
b. Increased heart rate.
c. Elevated BP.
d. Cyanosis of nail beds.

259.Liquid iron preparation, which of the following directions would be appropriate for the nurse to teach?
a. Administer at least one hour before meals.
b. Inform the patient about the loose stools.
c. Liquid iron must be taken with a straw.
d. Avoid juices with iron.

260.Which of the following can an 8-month old perform?


a. Walks up and down the stairs.
b. Climb out the crib.
c. Unscrew medicine bottles.
d. Put objects in the mouth.

261.Which of the following statements made by the parents show good understanding about the infant’s car
safety tips?
a. “If my baby is over 15 pounds, she could use a seat belt.”
b. “I should use “rear-facing car seat belt and put the baby in the back seat of the car.”
c. “Some provinces or states do not require infant car seat.”
d. “A rear-facing car seat can be used in the front seat if I have a car with air bags.”

262.A 45-year-old patient is overweight and would like to start a jogging program. When the nurse was
informed, the most appropriate advised would be:
a. The client should lose weight first.
b. Should have cholesterol checked.
c. Must undergo complete physical examination.
d. Should start walking before jogging.

263.Highest priority for the patient with a diagnosis of leukemia:


a. Implementation of reverse isolation.
b. Providing soft toothbrush.
c. Using the smallest gauge needle for IM injections.
d. Using razor blade when shaving.
264.Which of the following health teaching is appropriate for a teen-ager who is diagnosed with DM?
a. Increase his daily exercise.
b. Decrease his insulin dosage before exercising.
c. Increase food intake when exercising.
d. Carry food supplements during exercise day.

266.When teaching a mother about communicable diseases, the nurse informs her that chickenpox is:
a. Still communicable until all the vesicles have dried.
b. Still communicable even when just dried scabs remain.
c. No longer communicable after a high fever has subsided.
d. Not communicable as long as the vesicles are intact and surrounded by a red areola.

267.A child sustains a fractured femur in a bicycle accident. However, the admission x-ray films reveal
evidence of fractures of other long bones in various stages of healing. The nurse determines that this
child should be assessed for:
a. Child abuse
b. Vitamin D deficiency
c. Calcium deficiency
d. Potassium deficiency

268.The nurse is aware that discharge planning related to care of a child with Duchenne’s muscular
dystrophy should include teaching the parents about:
a. Range of motion exercises.
b. Maintaining a high caloric diet.
c. Instituting seizure precautions.
d. Restricting the use of larger muscles.

269.A 20-year-old female attending college is found unconscious in her dorm room. She has a fever and a
noticeable rash. She has just been admitted to the hospital. Which of the following tests is most likely
to be performed first?
a. Blood sugar
b. CT scan
c. Blood cultures
d. ABG

270.A 28-year-old male with history of diabetes has been found wandering around in a confused pattern.
The male is sweaty and pale. Which of the following tests is most likely to be performed fist?
a. Blood sugar
b. CT scan
c. Blood culture
d. ABG

271.A nurse has just started her rounds delivering medication. A new patient on her rounds is a 4-year-old
boy who is non-verbal. This child does not have on any ID. What should the nurse do?
a. Contact the provider
b. Ask the child to write his name on the paper.
c. Ask the co-worker about the ID of the child.
d. Ask the father who is in the room about the name of the child.

272.A 35-year-old male has been an insulin-dependent diabetic for five years and now is unable to urinate.
Which of the following would you most likely suspect?
a. Atherosclerosis
b. Diabetic neuropathy
c. Autonomic neuropathy
d. Somatic neuropathy
273.You are taking the history of a 14-year-old girl who has a BMI of 18. The girl reports of inability to
eat, induced vomiting and severe constipation. Which of the following would you most likely suspect?
a. Multiple sclerosis
b. Anorexia nervosa
c. Bulimia
d. Systemic sclerosis

274.A 24-year-old female is admitted to the ER for confusion. This patient has a history of a myeloma
diagnosis, constipation, intense abdominal pain, and polyuria. Which of the following would you most
likely suspect?
a. Diverticulosis
b. Hypercalcemia
c. Hypocalcemia
d. Irritable bowel syndrome

275.A new mother has some questions about PKU. Which of the following statements made by the nurse is
not correct regarding PKU?
a. A Guthrie test can check the necessity lab values.
b. The urine has a high concentration of phenylpyruvic acid.
c. Mental deficits are often present with PKU.
d. The effects of PKU are reversible.

276.A patient is getting discharged from a long term facility. The patient has a history of severe COPD and
PVD. The patient is primarily concerned about their ability to breathe easily. Which of the following
would be the best instruction for this patient?
a. Deep breathing techniques to increase O2 levels.
b. Cough regularly and deeply to clear airway passages.
c. Cough following bronchodilator utilization.

277.A nurse is caring for an infant that has recently been diagnosed with a congenital heart defect. Which
of the following clinical signs would most likely be present?
a. Slow pulse rate
b. Weight gain
c. Decreased systolic pressure
d. Irregular WBC lab values.

278.A nurse is administering blood to a patient who has a low hemoglobin count. The patient asks how
long the RBC last in the body? The correct response is:
a. The life span of RBC is 45 days.
b. The life span of RBC is 60 days.
c. The life span of RBC is 90 days.
d. The life span of RBC is 120 days.

279.A 65-year-old man has been admitted to the hospital for spinal stenosis surgery. When does the
discharge training and planning begin for this patient?
a. Following surgery.
b. Upon admit.
c. Within 48 hours.
d. Preoperative discussion.

280.A patient’s chart indicates a history of hyperkalemia. Which of the following would you not expect to
see with this patient if these conditions were acute?
a. Decreased HR
b. Paresthesias
c. Muscle weakness of the extremities
d. Migraines
281.A 78-year-old client is admitted for open reduction of a wrist fracture. That night she is found
wandering in the hallway near the fire alarm. When the nurse calls the client’s name, the client says, “If
you come any closer to me I’ll pull this fire alarm. You’re a drug addict. I saw you taking drugs out of
the cabinet. “ The best response by the nurse would be:
a. “I am concerned you’ll hurt your arm. I’ll ask someone to take you back to your room.”
b. “Older adults often become confused when they’re in a new place. I’ll help you back to your
room.”
c. “Please don’t pull the fire alarm. You’ll wake up everyone else.”
d. “You are in the City hospital. I am not a drug addict. I’ll escort you to your room.”

282.Miss Helen Roller is admitted following a diagnosis of congestive heart failure. She tells you she is
unable to wash herself without assistance because of shortness of breath. You notice she also has
difficulty walking to the bathroom unassisted for the same reason. Which of the following nursing
diagnosis is most important?
a. “Decreased cardiac output related to side effects of medications.”
b. “Activity intolerance related to decreased oxygenation.”
c. “Altered tissue perfusion related to impaired gas exchange.”
d. “Self care deficit related to congestive heart failure.”

283.Appropriate nursing instruction to the patient with a diagnosis of AIDS would include:
a. Cook your meat very well.
b. Avoid eating on the same table your family eats on.
c. Avoid crowds.
d. Use disposable cutleries.

284.A newly diagnosed HIV patient is in you care. The patient has a positive PPD result of 5 mm
induration and a normal CXR. Which of the following represents preferred treatment of this patient?
a. INH and pyridoxine
b. IHN and Rifampin
c. Rifampin and Streptomycin
d. No pharmacotherapeutic regimen necessary.

285.Mr. D was placed on tricyclic antidepressant for his depression. The patient demonstrates that he
understands the teaching plan by stating,
a. “I must exercise and eat foods high in fiber.”
b. “I must have my blood drawn at least once a month after discharge.”
c. “I must stay out of the sun or wear sun block.”
d. “I cannot eat cheese or drink wine.”

286.In caring for a patient with ALS (Amyotrophic Lateral Sclerosis), the nursing diagnosis with the
HIGHEST priority would be:
a. Impaired physical mobility.
b. Altered role performance.
c. Potential for ineffective airway clearance.
d. Potential for impaired verbal communication.

287.The physician orders non-weight bearing with crutches for a patient with a leg The nurse would
recommend which of the following herb to the patient with vomiting and nausea?
a. Ginger root
b. Echinacea
c. Ginko
d. Ginseng
288. A 55-year-old male patient was admitted with a diagnosis of BPH (benign prostatic hypertrophy).
Which of the following herbal medications, when asked by the patient, is appropriate alternative
treatment?
a. St John’s wort
b. Dong Quai
c. Saw Palmetto extract
d. Echinacea

289.A patient with rheumatoid arthritis asks the nurse about ways to decrease morning stiffness. The nurse
should suggest:
a. Wearing loose but warm clothing.
b. Avoiding excessive physical stress and fatigue.
c. Taking a hot tub bath or shower in the morning.
d. Planning a rest break periodically for about 15 minutes.

290.After a painful exacerbation of rheumatoid arthritis, a patient is to begin a walking and exercise
program. An appropriate outcome would be that the patient:
a. Avoids exercising when there is some discomfort.
b. Is pain free while engaging in the activity program.
c. Walks and exercise even when the pain is severe.
d. Exercise unless the discomfort becomes too great.

291.A patient with lower back pain is tentatively diagnosed as having a herniated intervertebral disc. When
assessing this patient’s back pain, the nurse should ask:
a. “Is there any tenderness in the calf of your leg?”
b. “Have you had any increase burning sensation on urination?”
c. “Do you have any increase in pain during bowel movement?”
d. “Does the pain begin in your flank and move around to the groin?”

292.A patient is awaiting surgery for a ruptured lumbar nucleus pulposus. The nurse’s teaching should
include that the pain will most likely increase if the patient:
a. Lies on the side.
b. Flexes the knees.
c. Coughs excessively.
d. Sits for long periods.

293.After a patient with multiple fractures of the left femur is admitted to the hospital for surgery, the
patient demonstrates cyanosis, tachycardia, dyspnea, and restlessness. Initially the nurse should:
a. Administer oxygen by mask.
b. Immediately call the physician.
c. Place the patient in the supine position.
d. Place the patient in the high-fowler’s position.

294.A patient with pain and paresis of the left leg is scheduled for an electromyography. Before the test, the
nurse should explain that:
a. The involved area will be shaved immediately before testing.
b. The patient’s heart rate and rhythm will be frequently monitored.
c. Needles will be inserted into the affected muscles during the test.
d. The patient will be kept in a recumbent position after the procedure.

295.The nurse recognizes that the most specific diagnostic test that the physician can perform for a patient
who is suspected of having myasthenia gravis is:
a. An electromyography (EMG).
b. The pyridostigmine (Mestinon) test.
c. The endrophonium chloride (Tensilon) test.
d. A thorough history and physical assessment.
296.A young female patient comes to the physician because she has been experiencing fatigue and double
vision. The physician suspects myasthenia gravis. When obtaining information from the patient, the
nurse would expect her to report that:
a. Her level of fatigue has been constant.
b. The longer she rests the weaker she feels.
c. Her strength increases with progressive activity.
d. The symptoms seem more severe in the evening.

297.When talking with a patient who has been diagnosed as having myasthenia gravis, the nurse observes
that the patient has:
a. Problem with cognition.
b. Difficulty swallowing saliva.
c. Intention tremors of the hands.
d. Non-intention tremors of the extremities.

298.The nurse notes that a patient exhibit the characteristic gait associated with Parkinson’s disease. When
recording on the patient’s chart, the nurse should describe this gait as:
a. Ataxic
b. Spastic
c. Shuffling
d. Scissoring

299.While performing the history and physical examination of a patient with Parkinson’s disease, the nurse
should assess the patient for:
a. Frequent bouts of diarrhea.
b. Hyperextension of the neck.
c. A low-pitched, monotonous voice.
d. A recent increase in appetite and weight gain.

300.The nurse might expect a patient with multiple sclerosis to complain about the most common initial
symptom which is:
a. Diarrhea
b. Headaches
c. Skin infections
d. Visual disturbances

301.Initially, after a cerebral vascular accident, a patient’s pupils are equal and reactive to light. Later the
nurse assesses that the right pupil is reacting more slowly than the left and the systolic blood pressure
is beginning to rise. The nurse recognizes that these adaptations are suggestive of:
a. Spinal shock
b. Hypovolemic shock
c. Transtentorial herniation
d. Increasing intracranial pressure

302.For proper foot care, the nurse should provide a patient with non-insulin dependent mellitus with
instructions to:
a. Remove all corns and stop smoking.
b. Always wear shoes and use natural fiber socks.
c. Wear nylon socks and wash feet in warm water.
d. Wear shoes that are slightly larger and avoid corn removers.

303.When preparing a patient for discharge, the nurse should teach the signs of hypothyroidism. The nurse
would be aware that the patient understands the teaching when the patient says, “I should call my
physician if I develop”
a. Dry skin and intolerance to cold.”
b. Muscle cramping and sluggishness.”
c. Fatigue and an increased pulse rate.”
d. Tachycardia and an increase in weight.”

304.The description that should be used for the soft swishing sounds of normal breathing heard when the
nurse auscultate a patient’s chest would be:
a. Fine crackles
b. Adventitious sounds
c. Vesicular breath sounds
d. Diminished breath sounds

305.When assessing a patient with pleural effusion, the nurse should expect to find:
a. Moist crackles at the back of the lungs.
b. Deviation of the trachea toward the involved side.
c. Reduced or absent breath sounds at the base of the lung.
d. Increased resonance with percussion of the involved area.

306.To best promote continued improvement in a patient’s respiratory status after chest drainage is
discontinued, the nurse should:
a. Continue observing for dyspnea and crepitus.
b. Encourage frequent coughing and deep breathing.
c. Turn the patient from side to side at least every 2 hours.
d. Encourage bedrest with active and passive range-of-motion exercises.

307.During auscultation of the heart, the nurse would expect the first heart sound (S1) to be the loudest at
the:
a. Apex of the heart.
b. Base of the heart.
c. Left lateral border.
d. Right lateral border.

308.Thrombus formation is a danger for all post operative patients. The nurse should act independently to
prevent this complication by:
a. Applying elastic stockings.
b. Massaging gently with lotion.
c. Encouraging in-bed exercises.
d. Providing adequate fluids intake.

309.The nurse recognizes that a pacemaker is indicated when a patient is experiencing:


a. Angina
b. Chest pain
c. Heart block
d. Tachycardia

310.A patient is suspected of having thrombophlebitis of the left lower extremity. During the initial
assessment the nurse should specifically observe the patient for:
a. Edema of the left leg.
b. Mobility of the left leg.
c. A positive left-sided babinski reflex.
d. Presence of peripheral arterial pulses.

311.The day before a patient who has had a subtotal gastrectomy is to be discharged, the patient complains
of perspiring and having epigastric discomfort about a half hour after eating lunch. The symptoms
disappear within a few minutes. The nurse recognizes that the teaching about prevention is understood
when the patient states, “I can limit these symptoms by:
a. increasing fluids with each meal.”
b. avoiding spicy, gas-forming meals.”
c. resting before and after each meal.
d. eating small, low carbohydrate meal.”

312.A patient with a tentative diagnosis of cholecystitis is discharged from the emergency room with
instructions to make an appointment for a more definitive diagnosis workup. The recommendation that
would produce the most valuable diagnostic information would be:
a. “Keep a journal related to your pain.”
b. “Save all stool and urine for inspection.”
c. “Follow the doctor’s orders exactly without question.”
d. “Keep a record of the amount of fluid you drink daily.”

313.The nurse would know that dietary teaching for a patient with a colostomy had been effective when the
patient states, “It is important that I eat
a. food low in fiber so that there is less stool.”
b. bland foods so that my intestines do not become irritated.”
c. everything I ate before the operation while avoiding foods that cause gas.”
d. soft foods that are more easily digested and absorbed by my large intestine.”

314.A patient has been receiving a cardiac glycoside, a diuretic, and a vasodilator and is on bedrest. The
patient’s apical pulse rate is 44. The nurse concludes that this pulse rate is most likely a result of the:
a. Diuretic
b. Vasodilator
c. Bedrest regimen
d. Cardiac glycoside

315.The physician prescribes Isordil 10 mg PRN TID and a nitroglycerin transdermal disc once a day for a
patient with chronic angina pectoris. The patient asks the nurse why the Isordil is prescribed. The
nurse’s best response would be”
a. “it prevents the blood from clotting.”
b. “It suppresses irritability in the ventricles.”
c. “It allows more oxygen to get to the heart tissue.”
d. “It increases the force of contraction of the heart.”

316.When obtaining the health history from a patient who is seeking contraceptive information, the nurse
should consider that oral contraceptives are contraindicated in a patient who:
a. Is over 30 years of age.
b. Smokes a pack of cigarettes per day.
c. Has a history of borderline hypertension.
d. Has had at least one multiple pregnancy.

317.The nurse knows that clients need a well balanced diet daily and possibly vitamin and iron
supplements during pregnancy to maintain their own and fetal needs during gestation. Folic acid may
also need to be supplemented. To obtain this nutrient, the nurse should tell pregnant clients to eat:
a. Celery
b. Bananas
c. Cooked beef liver
d. Boiled turkey breast

318.The nurses informs the pregnant client who does not like milk that there are other foods that are good
sources of calcium and advises the client to eat:
a. Corn
b. Liver
c. Broccoli
d. Lean meat
319.While having contractions every 2 to 3 minutes, which last from 60 to 70 seconds, a client complains
of having rectal pressure. The nurse should:
a. Attach an external fetal monitor.
b. Inspect the client’s perineum for bulging.
c. Determine when the client’s labor began.
d. Ask the client if her membranes have ruptured.

320.Thirty minutes after delivery the nurse finds that a mother’s uterus is relaxed and the lochia is
excessive. The fist action by the nurse should be to:
a. Check the mother’s vital signs.
b. Elevate the foot part of the bed.
c. Immediately notify the physician.
d. Massage the mother’s uterus and expel any clots.

321.After delivery, a mother tells the nurse, “I’m so cold and I can’t stop shaking.” The nurse should tell
the mother:
a. “I am going to take your blood pressure and pulse.”
b. “Let me check your fundus to see whether it is firm.”
c. “Please, turn on your side so I can check the amount of lochia.”
d. “I will put some warm blankets on you; the chill will subside soon.”

322.The best nursing intervention to minimize perineal edema following episiotomy would be:
a. Hot sitz bath.
b. Aspirin 10 grains po Q4H.
c. Ice packs to the perineum.
d. Elevation of hips on a pillow.

323.After I minute after birth, the nurse notes that an infant is crying, has a heart rate of 140, has
acrocyanosis, resists the suction catheter, and keeps arms extended. The nurse should assign this infant
an Apgar score of:
a. 4
b. 6
c. 8
d. 10

324.On the third day post partum day, a mother asks why her newborn’s skin has begun to appear yellow.
The nurse should plan to teach her that the change in skin tone is a result of:
a. Breast milk ingestion.
b. Inadequate fluid intake.
c. Breakdown of red blood cells.
d. An immature vascular system.

325.Although the exact cause of pregnancy-induced hypertension is unknown, the nurse knows that it is
often associated with:
a. A vitamin deficiency.
b. A limited amount of calories.
c. An inability to absorb minerals.
d. An inadequate intake of protein.

326.A mother indicates correct understanding of postpartum discharge instructions when starting that she
will take her iron supplements with:
a. Milk
b. Water
c. Each meal
d. Orange juice
327.An assessment of a 6-month-old infant’s growth and development level should reveal that the infant
can:
a. Say, “Mama”.
b. Crawl forward.
c. Turn pages in a book.
d. Hold a bottle without help.

328.If a 5 ½ - month-old infant’s immunizations are on schedule, the nurse can assume that the baby has
already received:
a. Measles, mumps, and rubella vaccines.
b. A booster dose of trivalent oral poliovirus vaccine.
c. Two doses of diphtheraia, tetanus, and pertussis vaccine.
d. The fist booster dose of diphtheria, tetanus, and pertussis vaccine.

329.If a 9-month-old’s immunization schedule is up to date, the next immunization that the infant should
receive at 12 to 15 months of age is:
a. Tetanus toxoid
b. Trivalent oral polio vaccine
c. Measles, mumps, and rubella vaccine
d. Diptheria, tetanus, pertussis vaccine.

330.In addition to the relief of pain, the nurse should direct the care for a patient with sickle cell crisis
toward:
a. Antibiotics and narcotic regulation.
b. Oxygenation and adequate hydration.
c. Hydration and physiologic counseling.
d. Oxygenation and Factor VIII replacement.

331.A child with rubella should be isolated from an unimmunized:


a. 20-year-old brother living at home.
b. 3-year-old girl friend that lives next door.
c. 12-year-old sister who has rubeola as a child.
d. 18-year-old female cousin who has recently married.

332. The nurse is aware that immunization of infants does not begin until 2 months of age because:
a. The neonatal spleen is unable to produce efficient antibodies.
b. Infants under 2 months are rarely exposed to infectious diseases.
c. The immunization would attack the immature infant’s body and produce the disease.
d. Maternal antibodies interfere with the development of active antibodies by the infant.

333.Understanding that there is a need to protect susceptible persons from exposure to chickenpox during
the acute phase, the nurse should question the mother of a child with chickenpox about relatives or
friends who are receiving:
a. Long-term anticonvulsant therapy.
b. Prolonged topical antibiotic therapy.
c. High doses of systemic steroid therapy.
d. Therapeutic doses of vitamins and minerals.

334.The nurse who is performing venipuncture, Guthrie test, on a newborn to assess for PKU is said to be
doing:
a. Health promotion.
b. Primary
c. Secondary
d. Tertiary
335.Health teachings on foods rich in folic acid on a pregnant mother to prevent complications on a
growing fetus is under:
a. Health promotion
b. Primary
c. Secondary
d. Tertiary

336.Screening for high blood cholesterol in the community is under:


a. Health promotion
b. Primary
c. Secondary
d. Tertiary

337.Health on the effects of smoking and lecture on how to stop smoking is under:
a. Health promotion
b. Primary
c. Secondary
d. Tertiary

338.Mr. Smith is 67-year-old black male brought to the hospital by his wife, who stated that he fell down
20 minutes ago and has been unable to speak or move his right side since then, Mr. Smith has no
significant past medical history. On exam, Mr. Smith is conscious, very anxious, his speech is garbed
and unintelligible, he has a left facial droop, and he is completely right hemiphlegic. The most likely
etiology for his symptoms is:
a. CVA
b. Traumatic brain injury
c. Brain tumor
d. Alzheimer’s disease

339.Jane is a 30-year-old white female who presents to the neurology clinic with 2-year-history of
progressive visual disturbance, emotional lability, impaired coordination, extremity weakness and
aching, and dizziness. She says that she notices the symptoms more on very warm days and just after
going to the gym. Given Jane’s history, the most likely diagnosis is:
a. Multiple sclerosis
b. Myasthenia gravis
c. Brain tumor
d. Exhaustion

340.Which is a characteristic of heart rate in atrial fibrillation (AF)?


a. Irregular ventricular rhtyhm
b. Fast and thready or bounding
c. Atrial very rapid 400 to 700 bpm
d. Irregular RR interval

341.Mrs. A is a 40-year-old woman who reports menstrual irregularities and believes she is experiencing an
early menopause. She reports feeling fatigue and restless at times. Physical findings reveal a thin
woman with fine hair, moist, warm skin, and a goiter with bruit present. Heart rate is 110; BP 140/80.
These findings are consistent with:
a. Hypothyroidism
b. Grave’s disease
c. Myxedema
d. Thyroid cancer

342.Physical findings commonly seen in hypothyroidism include:


a. Coarse hair, thin brittle nails
b. Malnourished appearance, alopecia
c. Tachycardia, hyperreflexia
d. Confusion, stupor

343. A patient is scheduled for below-knee-amputation of the right leg. Legally, the patient may not sign
the operative consent if:
a. Ambivalent feelings regarding the operation are present.
b. Any sedative type of medication has recently been administered.
c. A discussion of alternatives with two physicians has not occurred.
d. A complete history and physical have not been performed and recorded.

344. A male patient with thrombophlebitis is apprehensive about the possibility of a clot teaching his heart,
causing sudden death. The nurse’s initial intervention should be to:
a. Clarify his misconception.
b. Explain preventive measures.
c. Teach recognition of early symptoms.
d. Encourage discussion of the patient’s concern.

345. The patient weighed 210 pounds on admission to the hospital. After 2 days of diuretic therapy the
patient weighs 205.5 pounds. The nurse could estimate that the amount of fluid the patients has lost
is:
a. 0.5 L
b. 1.0 L
c. 2.0 L
d. 3.5 L

346. A patient was placed on a low sodium diet. The family asks if they can bring some snacks from home.
The nurse suggests that they bring foods low in sodium such as:
a. Ice cream
b. Celery sticks
c. Fresh oranges
d. Peanut butter cookies

347. The patient with ascites has a paracentesis and 1500 ml of fluid is removed. Immediately following
the procedure it is most important for the nurse to observe for:
a. A rapid, thready pulse
b. Decreasing peristalsis
c. Respiratory congestion
d. An increase in temperature

348. The patient has been receiving 2500 ml of IV fluid and 300 to 400 ml of oral intake daily for 2 days.
The patient’s urine output has been decreasing and now has been less than 40 ml per hour for the past
3 hours. The nurse should immediately:
a. Catheterize the patient to empty the bladder.
b. Assess breath sounds and obtain the patient’s vital signs.
c. Check for dependent edema and continue to monitor I/O.
d. Decrease the IV flow rate and increase oral fluids to compensate.

349. When taking the BP of a patient who has AIDS the nurse must:
a. Wear clean gloves.
b. Use barrier techniques.
c. Wear a mask and gown.
d. Wash the hands thoroughly.

350.When explaining the therapeutic regimen of vitamin B12 for pernicious anemia to a patient the nurse
should explain that:
a. Weekly Z-track injections are required for control.
b. Daily intramuscular injections are required for control.
c. Intramuscular injections once a month will maintain control.
d. Oral tablets of vitamin B12 taken daily will keep the symptoms under control.

351.When obtaining a health history from a young patient with probable acute lymphocytic leukemia
(ALL), the clinical manifestations the nurse should expect to be present:
a. Petechiae, alopecia
b. Anorexia, insomia
c. Anorexia, petechiae
d. Alopecia, bleeding gums

352.In a cardiovascular emergency, which arteries should you palpate to assess the patient’s pulse?
a. Radial and brachial arteries.
b. Brachial and femoral arteries.
c. Carotid and radial arteries.
d. Carotid and femoral arteries.

353.Which of the following shouldn’t be considered when assessing an elderly patient?


a. An elderly patient may have one or more chronic diseases.
b. Any new illness or physical problem is likely to cause a change in mental status because in elderly
patients the brain is the most vulnerable organ.
c. Most elderly patients have trouble hearing, so you’ll need to speak loudly when obtaining the
health history.
d. The elderly patient may have slowed intestinal motility and weakened bones and joints.

354.You’re able to elicit a Babinski’s response in your 9-month-old patient. This is


a. A normal finding.
b. An abnormal finding.
c. Requires consult with the physician.
d. Must be ignored.

355.Which two percussion sounds are normally heard over the abdomen?
a. Tympany and dullness.
b. Resonance and tympany.
c. Flatness and dullness.
d. Flatness and hyperresonance.

356. If a patient comes into facility with muscle weakness, where should you focus your physical
assessment?
a. Muscle tone
b. Muscle strength
c. ROM
d. Sensory function

357. Prolonged expiration is common among COPDs and it suggests which of the following?
a. Narrowed lower airways.
b. Narrowed upper airways.
c. Altelectasis.
d. Bronchial edema.

358. Which of the following indicate that the patient with Gullain Barre Syndrome is recovering?
a. BP 120/70, HR 98; RR 18.
b. Moves toes and fingers; RR 18.
c. Unable to wiggle toes; sensation intact.
d. BP 80/60; HR 100; RR 20.
359. A 4-year-old child is being discharge. What is the suitable car seat for the child?
a. Rear facing seat.
b. Front facing seat.
c. Booster seat with car seat belt.
d. Back seat.

Define the following terms:

360. Kussmaul breathing:


a. Deep, rapid breathing.
b. Slow, deep breathing.
c. Deep, irregular breathing.
d. Breathing pattern seen in acidosis.

361. Cheyne-stokes breathing:


a. Breathing seen in brain-injured patients.
b. Shallow, fast with apnea in between.
c. Deep, rapid breathing seen among patients with increased ICP.
362. Homonymous hemianopsia:
a. Half of the visual field of the affected eye is lost.
b. The pupil in the affected eye is always dilated.
c. Lost of vision on the affected eye.
d. The pupil in the affected eye is not constricting.

363. Ipsilateral pupil:


a. The pupil on the affected eye is always dilated.
b. The pupil on the affected eye is always constricted.
c. Lost of vision on the affected eye.
d. Total blindness.

364. Atherosclerosis:
a. Calcium plaque in the arteries
b. Fatty deposit in the vessels
c. Fatty deposit in large veins.
d. Calcium deposit in the capillaries.

365. Ateriosclerosis:
a. Calcium plaque in the arteries
b. Fatty deposit in the vessels
c. Fatty deposit in large veins
d. Calcium deposit in the capillaries.

366. Cardiac output:


a. Amount of blood ejected in the left ventricle.
b. Amount of blood ejected in the right ventricle.
c. Amount of oxygenated blood in the left ventricle.
d. Amount of blood that distends the left ventricle.

367. Paroxysmal nocturnal dyspnea:


a. Feeling of suffocation when in recumbent position.
b. Persistent dyspnea at night.
c. Inability to sleep unless in a sitting position.
d. Can breathe more easily when in an upright position.

368. Loss of peripheral vascular resistance:


a. Hypotension
b. Fluid in the circulation
c. Vasoconstriction
d. Spasm in the arteries

369. Venous return:


a. Deoxygenated blood
b. Oxygenated blood
c. Blood going to the left side of the heart
d. Blood leaving the heart

370. Venous drainage:


a. Deoxygenated blood coming from the upper part of the body.
b. Oxygenated blood going up to the upper part of the body.
c. Blood flowing through the carotid arteries.
d. Blood returning to the heart through the neck veins.

371. Glucocorticoids:
a. Cortisol
b. ADH
c. Aldosterone
d. Androgens

372. Polyuria:
a. Inability of the kidneys to concentrate urine
b. Frequent urination
c. Dribbling
d. Dysuria

373. Cushingoid appearance:


a. Excessive cortisol level
b. ADH deficiency
c. Poor skin turgor
d. Tachycardia

374. Steatorrhea:
a. Greasy stools
b. Loose, frequent bowel movement
c. Constipation
d. Melena

375. Flaccid:
a. Weakening of body parts.
b. Loss of movement of body part.
c. Contraction of body parts.
d. Loss of sensation of body parts.

376. Bradykenisia:
a. Difficulty initiating movement
b. Inability to move due to weakness
c. Stooped position
d. Coordinated movements

377. Hypophonia:
a. Loss of volume of speech.
b. High-pitch tone.
c. Difficulty swallowing.
d. Drooling.

378. Illusion:
a. Misperception of actual stimuli
b. False sensory perception not related to actual, external stimuli
c. False, fixed idea, arising out of a person's inner needs that cannot be changed by reasoning
d. Hearing voices.

379. The assessment of a patient that would be most indicative of diabetes insipidus is:
a. Increased blood glucose.
b. Low urinary specific gravity.
c. Elevation of blood pressure.
d. Decreased serum osmolality.

380. In an emergency the rapid adjustments made by the body are associated with the increased activity
of the:
a. Thyroid gland
b. Adrenal gland
c. Pituitary gland
d. Pancreas

381. In Cushing’s syndrome excessive amounts of glucocorticoids and mineralcortocoids will increase
the patient’s:
a. Urine output
b. Glucose level
c. Serum potassium
d. Immune response

382. When teaching an athletic teenage patient about nutritional intake, the nurse should explain that the
carbohydrate food that would provide the quickest source of energy is:
a. Glass of milk
b. Slice of bread
c. Chocolate candy bar
d. Glass of orange juice

383. A patient is to have gastric gavage. When the gavage tube is inserted, the nurse should place the
patient in the:
a. Supine position
b. Mid-fowler’s position
c. Low-fowler’s position
d. High fowler’s position

384. The maximum safe height at which the container of fluid can be held when administering an enema
is:
a. 30 cm (12 inches)
b. 37 cm (15 inches)
c. 45 cm (18 inches)
d. 66 cm (26 inches)

385.Three days after admission for CVA, a patient has a nasogastric tube inserted and is receiving
intermittent feedings. To best evaluate if a prior feeding has been absorbed, the nurse should:
a. Evaluate the intake in relation to the output.
b. Aspirate for a residual volume and reinstill it.
c. Instill air into the stomach while auscultating.
d. Compare the patient’s body weight to the baseline data.
386.The nurse understands that an acute attack of pancreatitis can be precipitated by heavy drinking
because:
a. Alcohol promoted the formation of calculi in the cystic duct.
b. The pancreas is stimulated to secrete more insulin than it can immediately produce.
c. The alcohol alters the composition of enzymes so they are capable of damaging the pancreas.
d. Alcohol increases enzyme secretion and pancreatic duct pressure and causes backflow of
enzymes into the pancreas.

387.Jaundiced patients are susceptible to postoperative hemorrhage because their blood does not clot
normally due to the fact that:
a. Excess bile salts in the blood inhibit synthesis of prothrombin in the liver.
b. Excess bile salts in the blood inhibit synthesis of vitamin K and prothrombin in the liver.
c. Decreased bile salts in the blood inactivate prothrombinase and prevent formation of thrombin
from prothrombin.
d. Lack of bile in the intestine causes inadequate vitamin k absorption, which causes inadequate
prothrombin synthesis by the liver.

388.A patient with cholelithiasis experiences discomfort after ingesting fatty foods because:
a. Fatty foods are hard to digest.
b. Bile flow into the intestine is obstructed.
c. The liver is manufacturing inadequate bile.
d. There is inadequate closure of the ampulla of vater.

389.The nurse assesses the patient with cholecystitis for the development of obstructive jaundice, which
would be evidenced by:
a. Inadequate absorption of fat-soluble vitamin K.
b. Light amber urine, dark brown stools, yellow skin.
c. Dark-colored urine, clay-colored stools, itchy skin.
d. Straw-colored urine, putty-colored stools, yellow sclerae.

390.Prior to a cholecystectomy,t he physician orders vitamin K. This is administered because it is used in


the formation of:
a. Bilirubin
b. Prothrombin
c. Thromboplastin
d. Cholecystokinin

391.The most therapeutic diet for a patient with hepatic cirrhosis would be:
a. High protein, low carbohydrate, low fat.
b. Low protein, low carbohydrate high fat, soft.
c. High carbohydrate, low saturated fat, 1200 calories.
d. Low sodium, protein to tolerance, moderate fat, high calorie, soft.

392.The nurse understands that the patient with esophageal varices should not be given food such as:
a. Crackers
b. Purred food
c. Liquid
d. Soft

393. The physician orders a paracentesis for a patient with ascites. Before the procedure, the nurse should
instruct the patient to:
a. Empty the bladder
b. Eat foods low in fat
c. Remain NPO for 24 hours
d. Assume the supine position
394. If intubation is indicated for a patient with bleeding esophageal varices, the type of tube most likely to
be used would be a (an):
a. Levin tube
b. Salem sump
c. Miller-Abbot tube
d. Sengstaken-Blakemore tube

395. The basic pathophysiologic problem in cirrhosis of the liver causing esophageal varices is:
a. Ascites and edema
b. Portal hypertension
c. Loss of regeneration
d. Dilated veins and varicosities

396.Following a CVA, a patient remains unresponsive to sensory stimulation. The lobe of the cerebral
cortex that registers general sensations such as heat, cold, pain, and touch is the:
a. Frontal lobe
b. Parietal lobe
c. Occipital lobe
d. Temporal lobe

397.To prevent toxoplamosis, the nurse should instruct to avoid:


a. Contact with cat feces
b. Working with heavy metals
c. Ingestion of fresh water fish
d. Excessive radiation exposure

398.The nurse should recognize that further teaching is needed when a patient with glaucoma states, “it
would be dangerous for me to:
a. use any sedatives.”
b. become constipated.”
c. use atropine in any form.”
d. release my emotions by crying.”

399.After a patient has cataract surgery, the nurse should:


a. Teach the patient coughing and deep breathing techniques.
b. Encourage eye exercises to strengthen the ocular musculature.
c. Keep the patient in supine position with head immobilize.
d. Advise patient to refrain from vigorous brushing of teeth and hair.

400.The primary responsibility of a nurse during a patient’s generalized motor seizure is:
a. Determining if an aura is experienced.
b. Inserting a plastic airway between the teeth.
c. Clearing the immediate environment for safety.
d. Administering the prescribed PRN anticonvulsant.

401.When a patient is unconscious, the nurse should expect the patient to be unable to:
a. Hear voices.
b. Control elimination.
c. Move spontaneously.
d. React to painful stimuli.

402. A patient regains consciousness and has expressive aphasia. As a part of the long-range planning, the
nurse should:
a. Provide positive feedback when the patient uses a word correctly.
b. Wait for the patient to verbally state needs regardless of how long it may take.
c. Suggest that the patient get help at home because the disability is permanent.
d. Help the family to accept the fact that the patient cannot participate in verbal communication.

403. Of the following combination of symptoms the most indicative of increased intracranial pressure is:
a. Weak rapid pulse, normal blood pressure, intermittent fever, lethargy.
b. Rapid weak pulse, fall in blood pressure, low temperature, restlessness.
c. Slow bounding pulse, rising blood pressure, elevated temperature, stupor.
d. Slow bounding pulse, fall in blood pressure, temperature below 97°F, stupor.

404. Patients with myasthenia gravis, Gullain-Barre syndrome, or amyotrophic lateral sclerosis experience:
a. Progressive deterioration until death.
b. Increased risk of respiratory complications.
c. Deficiencies of essential neurotransmitters.
d. Involuntary twitching of small muscle groups.

405. A nurse finds the victim under the wreckage of a collapsed building. The individual is conscious,
breathing satisfactorily, and lying on the back complaining of pain in the back and an inability to
move the legs. The nurse should first:
a. Leave the individual lying on the back with instructions not to move and then go seek additional
help.
b. Gently raise the individual to a sitting position to see if the pain either diminishes or increases in
intensity.
c. Roll the individual onto the abdomen, place a pad under the head, and cover with any material
available.
d. Gently lift the individual onto a flat piece of lumber and, using any available transportation, rush
to the closest medical institution.

Case Study:

Marie is a 5 - year - old girl is admitted with a diagnosis of Acute Lymphoblastic Leukemia . She enters
complete remission with intensive induction of chemotherapy, but she relapses after 3 months of
maintenance of chemotherapy with vincristine, prednisone, methotrexate and mercaptopurine. She develops
a neutropenia from the chemotherapy.

On admission, her lab results came back with WBC 15000, Hemoglobin 8 mg/dl; platelet 94,000. She
complains of ache all over her body, her gum bleeds very easily. Vital signs include BP 70/40; HR 130, RR
28; Temp 38ºC.

Questions:

406. Which of the following are consistent with Leukemia?


a. Weakness and fatigue.
b. Dizziness and vomiting
c. Bruising and petechiae
d. Light headedness and nausea

407. Incidence of Leukemia is higher in children. Pathophysiology of infection in Leukemia is due


primarily to:
a. Anemia secondary to blood loss.
b. Platelet count of 400,000 cu mm.
c. WBC of 20,000 cu mm.
d. Poor nutrition.

408.Neutropenia means
a. Decreased level of WBC, decreased platelet count, and decreased RBC.
b. Increased level of WBC, decreased platelet count, increased haemoglobin.
c. Elevated eosinophil level, elevated neutrophil, and elevated basophil.
d. Decreased hematocrit, decreased haemoglobin, and decreased RBC count.

409. White blood cells (WBC) play important role in resolving infection. Which of the following are the
components of WBC?
a. Neutrophil, Basophil, Eosinophil.
b. Thrombocytes, Leukocytes, Erythrocytes.
c. Platelets, Hematocrit, Immunoglobulins.
d. Granulocytes, Lymphocytes, Stem cells.

410.Which of the following nursing interventions would be contraindicated to the patient with Leukemia?
a. Take rectal temperature.
b. Use soft toothbrush.
c. Use normal saline for mouthwash TID.
d. Avoid using dental floss.

411.Which of the following indicate that the nurse understands the plan of care for Marie?
a. Use gauge #25 when giving IM injection.
b. Implement reverse isolation.
c. Rest is of vital importance.
d. Protect the child from people with respiratory infection.

412.When assessing an individual with a spontaneous pneumothorax, there is a danger of mediastinal shift.
Mediastinal shift is described as:
a. The tracheal deviation to the unaffected lung
b. Collapsed of the lung on the affected chest.
c. Deviation of the heart and the great vessels towards the affected lung.
d. Increased intrathoracic pressure.

Case Study

John H is a 66-year-old man with a history of heavy smoking presented himself to the ER due to difficulty
breathing of 2 years duration. He was prescribed some breathing medication by his family physician.
However, for the last two months he noted progression of the severity of symptoms and new medications
were prescribed. A week prior to admission, his difficulty in breathing worsened. There was non-productive
cough, easy fatiguability and basilar rales. He was diagnosed with respiratory problems with acute
exacerbation and stemming pneumonia. He was started on antibiotics, IV solumedrol Q6 hourly, oxygen at
2L/NC; bed rest; diet as tolerated; and ABG.

Mr. H was also diagnosed with effusion of the right lung. He is now scheduled for chest tube insertion. He
has 2 chest tubes. One is on the upper chest and the other tube was inserted on the lower chest.

Questions

413.What prevents air from entering the chest space when a patient has a chest tube?
a. The water seal chamber of the chest drainage device.
b. The patient’s ability to cough and breathe deeply.
c. The amount of suction applied to the chest tube.
d. The patient’s ability to remain immobile.

414. What is most likely occurring when persistent bubbling exist in the water seal chamber of a chest
drainage device?
a. There is an excessive amount of wall suction.
b. The amount of suction needs to be increased.
c. The patient’s chest tube is probably kinked.
d. There is an air leak in the chest tube or pleural cavity.
415. At what level is wall suction control generally set at?
a. -20 cm H2O
b. -40 cm H2O
c. -60 cm H2O
d. -80 cm H2O

416. The level of the water in the water-seal chamber is at


a. 2 cm
b. -20 cm
c. -10 cm
d. -80 cm

417.The nurse appropriately removes obstruction in the chest tube by which of the following?
a. Stripping the chest tube.
b. Milking the chest tube.
c. Stripping the tube may decrease the intrathoracic pressure.
d. Milking the tube requires squeezing its length without releasing it.

418.Important patient teaching on chest tube due to pneumothorax will include


a. inform the patient that bubbling is expected in the water-seal bottle.
b. inform the patient that there will be no drainage.
c. inform the patient that stripping may be done for tube obstruction.
d. inform the patient that continuous bubbling may be observed in the drainage bottle.

419.Negative pressure in the pleural space is indicated by


a. negative pressure in the suction-control chamber.
b. negative pressure in the water-seal chamber.
c. positive pressure in the water seal chamber.
d. bubbling in the water seal chamber.

420.In the context of Mr. H’s chest tubes, part of his routine assessment include:
a. Perform ABG testing.
b. Palpate around the chest tube site and assess for presence of crepitus.
c. Measure and empty the collection bottle at least at the end of the shift.
d. Encourage deep breathing and coughing exercises every 2 to 3 hours.

421.The nurse is caring for Mr. H. On the second postoperative day, the chest tube accidentally disconnects
from the drainage tube. The first action the nurse should take is
a. reconnect the tube
b. raise the collection chamber above the client's chest
c. remove the tube and replace it with new, sterile tube.
d. clamp the chest tube

422.Appropriate patient teaching when the chest tube is removed:


a. Instruct the patient take deep breath and hold it during removal.
b. Inform the patient that this is not a painful procedure.
c. Ensure that the site is covered with a loose, dry dressing.
d. Expect tachypnea after the removal.

423.To best promote continued improvement in a patient’s respiratory status after chest drainage is
discontinued, the nurse should:
a. Continue observing for dyspnea and crepitus.
b. Encourage frequent coughing and deep breathing.
c. Turn the patient from side to side at least every 2 hours.
d. Encourage bedrest with active and passive range-of-motion exercises.
424.The client with COPD has orthopnea. The nurse is aware that one of the following is a correct
description of orthopnea:
a. The patient has respiratory distress.
b. The patient is experiencing mucus plug in the airways.
c. The patient has periods of apnea.
d. The patient can breathe more easily when in sitting position.

425.The patient with pancreatitis may exhibit Cullen’s sign on the physical examination. Which of the
following symptoms best describes Cullen’s sign?
a. Jaundiced sclera.
b. Pain that occurs with movement.
c. Bluish discoloration of the left flank area.
d. Bluish discoloration of the periumbilical area.

426.The client who’s developing signs and symptoms indicate hyperglycemia?


a. Polydipsia, polyuria, and weight loss.
b. Weight gain, tiredness, and bardycardia.
c. Irritability, diaphoresis, and tachycardia.
d. Diarrhea, abdominal pain, and weight loss.

427.The most common cause of dementia in older adult is


a. Alcoholism
b. Vascular dementia
c. Parkinson’s disease
d. Alzheimer’s disease

428.The problem with sensory recognition is called


a. Aphasia
b. Apraxia
c. Agnosia
d. Dysarthia

429.Teach family members caring for a patient with dementia to


a. Avoid verbal cuing when he’s performing tasks.
b. Change daily routines regularly.
c. Provide clothes with zippers and buttons.
d. Foster familiarity in the patient’s environment.

430.Which of the following is correct about DIC?


a. It was once known as hemophilia A.
b. It’s both a bleeding and thrombotic disorder.
c. It’s clinically insignificant syndrome.
d. It’s never a complication of another condition.

431.Thrombosis in DIC is most likely to affect:


a. The common iliac artery
b. The subclavian artery
c. The descending aorta
d. Cutaneous capillaries

432.Which of the following lab finding is a strong indicator of DIC?


a. An elevated platelet count.
b. An elevated D-dimer.
c. A normal prothrombin time.
d. An elevated fibrinogen level.
433.A patient returned from the post anesthetic care unit with a nasogastric in place. The health care
provider’s order states “irrigate NG tube Q 4H. Which solution is the best irrigant?
a. 0.9% NS
b. Distilled water
c. Sterile water
d. Tap water

434.A patient who sustained massive internal injuries in a car crash becomes hypotensive and develops
acute renal failure. Which acid-base imbalance is he most likely to experience?
a. Respiratory acidosis
b. Metabolic acidosis
c. Respiratory alkalosis
d. Metabolic alkalosis

435.A patient who suffered burns 48 hours ago is entering the second phase of burn injury. What
physiologic changes can you expect?
a. Edema development
b. Profuse urination
c. Decrease in hemoglobin level
d. Increase WBC

436.A 42-year-old man with end-stage acquired immunodeficiency syndrome (AIDS) has frequent episodes
of watery stool. He’s nauseated and refuses to drink fluids. His body temperature is 102°F (38.9°C),
his BP is 82/52, and his pulse is 112. He has 0.9% NS infusing at 150 ml/hour through a large-bore
catheter. Which type of fluid is 0.9% NS?
a. Isotonic
b. Hypotonic
c. Hypertonic
d. Allotonic

437.The first compensatory mechanism for lost blood volume in children are
a. Increased HR and peripheral vasodilation.
b. Decreased HR and peripheral vasodilation.
c. Increased HR and peripheral vasoconstriction.
d. Decreased HR and peripheral vasoconstriction.

438.The most sensitive and easily quantifiable measure of circulatory status in a child is
a. Skin color
b. Pulse amplitude
c. Capillary refill
d. Skin temperature

439.Which capillary refill time is considered normal in a child?


a. 1 second
b. 2 seconds
c. 4 seconds
d. d. 5 seconds

440. A patient with Alzheimer’s disease is admitted with suspected dehydration after his daughter reports
that he’s refused to drink anything for the past 3 days. The health care provider orders several lab
tests. Which lab result is most expected with dehydration?
a. Urine specific gravity of 1.005.
b. Serum sodium level of 150 mEq/L.
c. Hematocrit of 38%
d. Elevated WBC
441. A 78-year-old patient is admitted with pulmonary edema and given I.V. morphine sulfate. Why?
a. To increase his blood pressure.
b. To promote diuresis.
c. To slow his breathing.
d. To put him to sleep.

442.During examination of a 36-year-old woman’s right breast, you palpate a lump. Which characteristic
most suggests that the lump may be malignant?
a. Softness
b. Irregular shape
c. Mobility
d. Tender

443.If your patient is in pain most of the day, administer analgesic doses
a. P.R.N.
b. At regularly scheduled time.
c. As soon as the last dose wears off.
d. Whenever pain returns, up to several times a day.

444.Pulsus Paradoxus is best described as


a. A difference in systolic and diastolic BP.
b. A difference of more than 10 mmHg in systolic arterial BP between inspiration and expiration.
c. A difference between diastolic BP between inspiration and expiration.
d. A difference in radial and apical pulse between inspiration and expiration.

445.A 52-year-old patient who underwent a right-sided thoracotomy 2 days ago complains of nausea. You
perform an abdominal assessment. Which sound should you hear when percussing over a dense tissue?
a. Tympany
b. Dullness
c. Flatness
d. Resonance

446.You’re filling in as a substitute school nurse. When an 11-year-old child with an earache and sore
throat reports to the nurse’s office, you inspect the tympanic membrane using an otoscope. Which
color suggests a normal eardrum?
a. Red
b. White
c. Gray
d. Yellowish

447.You’re performing an otoscopic examination on a 6-year-old child who comes to the nurse’s office
with an earache and a fever. In which direction should you pull his auricle to straighten the ear canal?
a. Down and forward
b. Up and forward
c. Up and back

448.Mr. Leon Garnett, 45-years-old, was admitted with pneumococcal pneumonia & Tuberculosis. Which
of the following nursing action is appropriate?
a. maintain a negative air pressure in the patient’s room
b. observe strict reverse isolation
c. limit patient’s contact especially when he is highly contagious
d. the patient is required to wear mask when in direst contact with visitors and care providers

449.In Rh incompatibility, the father is Rh positive and the mother is Rh negative. Rhogam must be
administered:
a. after abortion
b. as soon as mother gets pregnant
c. before the mother gets pregnant
d. one time immunization

Case Study:

This is a 70-year-old male patient admitted due to hiccups, confusion, right arm & right leg weakness,
facial droop and slurred speech. He has an Hx of CAD & had the 2nd CABG about 4 years ago. He was
being followed up by his cardiologist. Last night he went to bed @ 11:30 pm, fell asleep and nothing
unusual happened as noted by wife. However, when the patient woke up around 2 am, his wife noticed
changes as described above. His wife called their daughter who lives next door & then 911 was called.
Most of the symptoms have disappeared when the ambulance arrived. He was taken to CVH for further
evaluation & Tx.

Questions:

450.The nurse is aware that clinical manifestations differentiate right CHF from left CHF. Symptoms of
right CVA would include:
a. distended jugular veins
b. orthopnea
c. pink, frothy sputum
d. crackles

451.The nurse who works in the emergency department is assigned to the following clients. A well-
informed nurse knows that prompt Tx is indicated to which of the following client?
a. head injury
b. ventricular fibrillation
c. penetrating abdominal wound
d. a client who came in with 60% partial thickness burn

452.Ms. L had a C-section done. She delivered a healthy baby boy. On her 1st post operative day, Ms. L’s
roommate called the nurse & reports that Ms. L was very anxious & pale looking. Other clients were
in Ms. L’s room trying to help out. Upon assessment, her BP was 80/60, HR 110bpm. The top nursing
priority includes:
a. monitor the patient’s v/s & notify the doctor stat
b. Clear the patient’s immediate environment & ask other clients to move away
c. Place the patient in flat position and check her abdominal dressing.
d. Get the crash cart in anticipation for cardiac arrest

453.A patient has an anaphylactic reaction within the 1st half hour after an IV infusion containing
Ampicillin is started. The nurse understands that the symptoms occurring during an anaphylactic
reaction are the result of:
a. respiratory depression and asystole
b. constriction of the vessels and decreased CO
c. bronchial constriction & decreased peripheral resistance
d. decreased CO & dilation of major vessels

454.When providing discharge teaching to a family with AIDS, the nurse should teach the family:
a. “You need to boil the dishes for 30 minutes after use.”
b. “Let the pt eat from paper plates and discard them.”
c. “Wash the dishes in hot soapy H2O as you usually do.”
d. “Let the dishes soak in hot H2O overnight before washing.”

455.A patient with CAD has a sudden episode of cyanosis & a change in respiration. The nurse starts O2
administration immediately. In this situation:
a. O2 had not been ordered and therefore should not be administered
b. The nurse’s observation were sufficient to begin administration of O2
c. The symptoms were too vague for the nurse to diagnose a need for O2
d. The doctor should have been called for an order before O2 was began

456.The nurse would expect an elderly client with a hearing loss caused by aging to have:
a. ear impaction with cerumen
b. difficulty hearing women’s voices
c. Hx of middle ear infection
d. damage to the inner ear

457.Boy Buffington sustained an extensive 3rd degree burn on his right leg and right groin. During the 1st
24 hours, important Nursing care for the pt would include which of the following?
a. control of infection
b. nutrition
c. fluids resuscitation
d. provide adequate rest
458.Hirschprung’s Disease is somewhat similar to cystic fibrosis. In both diseases, the newborn does not
pass out meconium. Pathophysiologically, in Hirschprung’s Disease, the child’s affected organ is:
a. Small intestine
b. Large intestine
c. Stomach
d. Liver

459.Which of the following predisposing factors causes anemia among young adults?
a. sleeping pattern
b. not eating meat & vegetables
c. WBC 9000
d. drinking too much alcoholic beverage.

460.Which of the following signs and symptoms would a client with tuberculosis (TB) have?
a. Chest and lower back pain.
b. Chills, fever, night sweats, and hemoptysis.
c. Fever more than 104°F (40°C) and nausea.
d. Headache and photophobia.

461.Which of the following types of diet is implicated in the development of diverticulosis?


a. Low-fiber diet.
b. High-fiber diet.
c. High-protein diet.
d. Low-carbohydrate diet.

462.An elderly patient in the long-term facility expresses feeling lonely especially during the day. The staff
nurse who is assigned to the patient will appropriately do which of the following?
a. Provide unlimited visiting privileges to the family and encourage them to spend more time with
the patient.
b. Try to cheer the patient up by telling him about the beautiful, sunny day.
c. Encourage the patient to join the daily group activities held in the patient’s lounge.
d. Request other patients to visit the patient.

463.While cleaning the house, Glenda, G2P1 and 38 weeks pregnant, felt a sudden gush of clear fluids
from her vagina. She drives herself to the nearby community hospital. Upon admission, nursing
assessment confirms rupture of her bag of water. Which of the following should be a priority in the
care of the patient?
a. Monitor mother’s BP.
b. Monitor mother’s HR.
c. Monitor fetal heart tones.
d. Minimize internal examination.

BONUS
The nurse is caring for a patient in the ICU who has had a spinal cord injury. She observes that his last
blood pressure was 100/55, and his pulse is 48. These have both trended downwards from the
baseline. What should the nurse expect to be the next course of action ordered by the physician?
Assess the patient for decreased level of consciousness
Administer Normal Saline
Insert an NG Tube
Submit
Connect and read an EKG
Administer Normal Saline
Correct – The patient is entering neurogenic shock. Normal saline will replace fluid volume, treating
the hypotension and bradycardia symptomatically. Atropine sulfate is also commonly used to increase
the heart rate.
A nurse is caring for a patient who is suspected to have sustained a spinal cord injury. What best
describes the overarching principles used to guide the care for this type of condition?
Immobilize the cervical area to prevent further injury
Monitor the patient’s level of consciousness to prevent neurologic deterioration
Help the patient with activities of daily living and provide emotional and physical support to help
them adjust to their injury
Facilitate tissue perfusion to the spinal cord while maintaining airway and breathing
Submit

Correct – Maintaining airway, breathing, and circulation is both essential and guides the overall plan
of care for a patient with a spinal cord injury.
A 23-year-old woman is admitted to the infusion clinic after a Multiple Sclerosis Exacerbation. The
physician orders methylprednisolone infusions (Solu-Medrol). The nurse would expect which of the
following outcomes after administration of this medication?
A decrease in muscle spasticity and involuntary movements
A slowed progression of Multiple Sclerosis related plaques
A decrease in the length of the exacerbation
A stabilization of mood and sleep
Submit

A decrease in the length of the exacerbation


Correct – A methylprednisolone infusion is the first line of treatment during an acute exacerbation and
is used to decrease the length and severity of a relapse.
A nurse knows that which of these patients are at greatest risk for a stroke?
A 60-year old male who weighs 270 pounds, has atrial fibrillation, and has had a TIA in the past.
a 75-year old male who has frequent migraines, drinks a glass of wine every day, and is Hispanic.
A 40-year old female who has high cholesterol and uses oral contraceptives
A 65-year old female who is African American, has sickle cell disease and smokes cigarettes.
Submit

A 60-year old male who weighs 270 pounds, has atrial fibrillation, and has had a TIA in the past.
Correct – Common risk factors for developing stroke include: Atrial fibrillation, arteriosclerosis,
previous stroke or ischemic attack, heart surgery, valvular heart disease, diabetes, smoking, substance
abuse,obesity, sedentary lifestyle, oral contraceptive use, genetic tendency, migraines, older age, male,
African American/Hispanic/American Indian, Sickle Cell Anemia, and brain trauma. This man has the
greatest risk based on these risk factors.
A nurse frequently treats patients in the 72-hour period after a stroke has occurred. The nurse would be
most concerned about which of these assessment findings?
INR is 3 seconds long
Heart rate is 110 beats per minute
Intracranial Pressure is 22 mm/Hg
Blood pressure is 140/80
Submit

Intracranial Pressure is 22 mm/Hg


Correct – The patient is at greatest risk for an increased ICP resulting from edema 72 hours after a
stroke. A target ICP should be less than or equal to 15-20 mm/Hg
A nurse is caring for a patient scheduled to have cataract surgery. The patient asks why they developed
cataracts and how they can prevent it from happening again. What is the nurse’s best response?
“Age is the biggest factor contributing to cataracts.”
“Unprotected exposure to UV lights can cause cataracts”
“Age, eye injury, corticosteroids, and unprotected sunlight exposure are contributing factors to
cataracts.”
“Unfortunately, there is really nothing you can do to prevent cataracts, but they are amongst the
most easily treated eye conditions.”
Submit
“Age, eye injury, corticosteroids, and unprotected sunlight exposure are contributing factors to
cataracts.”
Correct – This answer covers the most common contributing factors for cataracts and includes
preventable risk factors.
A nurse is educating a patient about bimatoprost (Lumigan) eyedrops for the treatment of Glaucoma.
Which of the following indicates that the patient has a correct understanding of the expected outcomes
following treatment?
“I should be experiencing less blurriness in my central field of vision”
“This medication won’t help my vision at all, but will keep it from getting worse.”
“My peripheral vision should be increasing back to its normal state, but will take a few weeks to
do so.”
“This medication will help my eye restore intraocular fluid and increase intraocular pressure”
Submit

“This medication won’t help my vision at all, but will keep it from getting worse.”
Correct – Glaucoma cannot be cured, just treated. Treatment revolves around preventing further
deterioration.
A patient with Glaucoma is verbalizing his daily medication routine to the nurse. He states he has two
different eyedrop medications, both every twelve hours. He washes his hands, instills the drops, closes
his eyes gently, and presses his finger to the corner of his eye nearest his nose. After waiting 1 minute
with his eyes closed, he instills the other medication in the same way. What is the nurse’s best
response?
“You should wait more than 1 minute between different medications.”
“Your routine is very good! Can you demonstrate it for me?”
“It is actually not the best practice to close your eyes after instilling eyedrops.”
“You should actually be pressing your finger in the other corner of the eye.”
Submit

“You should wait more than 1 minute between different medications.”


Correct – It is recommended to wait 10-15 minutes between different eyedrop medications to give
them time to absorb an avoid one medication washing another one out.
A nurse would evaluate which of these patients as appropriate candidates for a closed MRI without
contrast, based on the information given?
A 20-year old woman who has unexplained joint pain and a low BMI.
A 35-year old woman with Multiple Sclerosis and has been trying to conceive.
A 67-year old man who has had an open-heart surgery 4 years ago.
A 40-year old woman who has been in a hypomanic state for the last 2 days.
Submit

A 20-year old woman who has unexplained joint pain and a low BMI.
Correct – MRI can be used to diagnose musculoskeletal disorders, and this patient has no
contraindications to an MRI.
A nurse is caring for a patient in the cardiac care unit who is taking bumetanide (Bumex) and is
diagnosed with Parkinson’s Disease. An unlicensed assistive personnel is assisting with feeding the
patient. Which of these foods would the nurse stress for the patient to eat most?
Foods containing the least amount of salt
Foods containing the most amount of potassium
Foods containing the most amount of calories
Foods containing the most amount of fiber
Submit

Foods containing the most amount of potassium


Correct – Bumex is a loop diuretic and can cause hypokalemia. Ensuring potassium is included in the
diet is a priority and can directly avoid a hypokalemic crisis..
A nurse knows that which of these patients are at greatest risk for a developing osteoporosis?
An 80-year old man who has a thin build
A 48-year old african american female who smokes cigarettes and drinks alcohol
A 55-year old female with an estrogen deficiency
A 70-year old caucasian female who takes oral corticosteroids
Submit

A 70-year old caucasian female who takes oral corticosteroids


Correct – This patient has by far the most risk factors, 3 of which are primary and one secondary. Age,
gender, ethnicity are three primary risk factors, while her corticosteroid treatment is the secondary risk
factor, bringing her total up to four.
A 30-year old Caucasian woman who works the night shift has been found to have early bone loss and
has a high risk for osteomalacia and bone degradation. She asks the nurse exactly why she should take
Vitamin D supplements. What is the nurse’s best response?
“It’s a standard part of the overall nutritional treatment for the prevention of osteomalacia”
“It helps your intestines absorb calcium, which is important for bone formation.”
“It stimulates skin cells to produce calcium, which is then released into the bloodstream to be
used for bone formation.”
“Vitamin D supplements should not be taken by someone of your age.”
Submit

Correct – This is the correct mechanism of action for Vitamin D


“It stimulates skin cells to produce calcium, which is then released into the bloodstream to be used for
bone formation.”
A nurse is caring for a patient with a cast on the right leg. Which of these assessment findings would
most concern the nurse?
The capillary refill time is 2 seconds
The patient complains of itching and discomfort
The cast has a foul-smelling odor
The patient is on antibiotics
Submit

The cast has a foul-smelling odor


Correct – A foul-smelling odor is a sign of infection or a pressure ulcer within the cast. Other
symptoms include a feeling of warmth, tightness and pain.
A nurse is orally administering alendronate (Fosamax), a bisphosphonate drug. The patient is largely
bed-bound and being treated for osteoporosis. What nursing consideration is most important with
administration of this drug?
Sit the head of the bed up for 30 minutes after administration
Give the patient a small amount of water to drink.
Feed the patient soon, at most 10 minutes after administration
Assess the patient for back pain or abdominal pain
Submit

Sit the head of the bed up for 30 minutes after administration


Correct – Bisphosphonates are associated with esophageal irritation that can lead to esophagitis.
Sitting upright decreases the time the medication spends in the esophagus.
A nurse is asked by a patient to describe in layman’s terms an overview of the condition called
osteomyelitis. What would be the nurse’s best response?
“Osteomyelitis is a gradual breakdown and weakening of your bones. It’s most often age-
related.”
“Osteomyelitis is caused by not having enough Vitamin D, which in turn causes a your bones to
be softer and de-mineralized.”
“Osteomyelitis is an infection in the bone. It can be caused by bacteria reaching your bone from
outside or inside your body.”
“This is a question that should be directed to your Healthcare Provider.”
Submit

Correct – This appropriately explains osteomyelitis


The infection control nurse is assigned to a patient with osteomyelitis related to a heel ulcer. The
wound is 5cm in diameter and the drainage saturates the dressing so that it must be changed every
hour. What is her priority intervention?
Place the patient under contact precautions
Use strict aseptic technique when caring for the wound
Place another dressing to reinforce the first one
Elevate the patient’s leg to prevent more drainage
Submit

Place the patient under contact precautions


Correct – A patient with an infectious wound, especially one not adequately contained by a dressing,
should be put under contact precautions.
A nurse in the emergency room receives a patient who had his left elbow fractured in a fight. He had
waited 5 hours before coming to the emergency room. His left hand has an unequal radial pulse, is
swollen, and is numb and tingling. What is the nurse’s priority intervention?
Place the patient in a supine position
Ask the patient to rate his pain on a scale of 1 to 10.
Wrap the fractured area with a snug dressing
Start an IV in the other arm.
Submit

Start an IV in the other arm.


Correct – Starting an IV is a nursing priority prior to emergency surgery. The patient may be in the late
stages of Acute Compartment Syndrome and may need a fasciotomy, in which the surgeon relieves
pressure by making an incision into the affected area.
A nurse is caring for a female patient 24 hours after a hip fracture. The patient is on bedrest. The nurse
knows that which regular assessment or intervention is essential for detecting or preventing the
complication of Fat Embolism Syndrome?
Performing passive, light, range of motion exercises on the hip as tolerated.
Assess the patient’s mental status for drowsiness or sleepiness.
Assess the pedal pulse and capillary refill in the toes.
Administer a stool softener as ordered
Submit

Correct – A decreased Level of Consciousness is the earliest sign of FES, caused by decreased oxygen
level.
What is the overarching nursing concern when caring for patients being treated with splints, casts, or
traction?
To assess for and prevent neurovascular complications or dysfunction
To ensure adequate nutrition during the healing process
To provide patient education for maintenance of splints, casts, or traction in the community.
To treat acute pain
Submit

To assess for and prevent neurovascular complications or dysfunction


Correct – This is the priority nursing diagnosis for patients with extremity fractures.
What nursing action demonstrates the nurse understands the priority nursing diagnosis when caring for
patients being treated with splints, casts, or traction?
The nurse assesses extremity pulse, temperature, color, pain, and feeling every hour.
The nurse orders meals with adequate protein and calcium for the patient.
The nurse teaches the patient never to insert objects under a cast to scratch an itch.
The nurse administers oral painkillers as ordered
Submit

The nurse assesses extremity pulse, temperature, color, pain, and feeling every hour.
Correct – The priority nursing diagnosis would be Risk for Peripheral Neurovascular Dysfunction
related to fractures, which is demonstrated by this action.
A patient is admitted and complains of gastric pain, fever, and diarrhea. Which assessment finding
should be reported to the healthcare provider immediately?
Abdominal distention
A bruit near the epigastric area
3 episodes of vomiting in the last hour
Blood pressure of 160/90
Submit

A bruit near the epigastric area


Correct – A bruit in the aortic area signals the presence of an aneurysm. This is life-threatening and
must be reported immediately.
The nurse in the day surgery center cares for a patient who has undergone an endoscopic procedue
with general anesthesia. The nurse understands that which nursing consideration is a priority
immediately after an endoscopic procedure?
Raise the siderails of the patient bed
Do not offer fluids, food or any oral intake
Check the temperature of the patient
Teach the patient to avoid aspirin or NSAIDS
Submit

Do not offer fluids, food or any oral intake


Correct – Endoscopies involve passing a tube through the mouth into the esophagus or upper GI.
Anesthesia is often given to inactivate the gag reflex, making the patient vulnerable to aspiration.
A nurse is preparing to palpate and percuss a patient’s abdomen as part of the assessment process.
Which of these findings would cause the nurse to immediately discontinue this part of the assessment?
The patient states “That sounds like it might hurt me.”
There is a pulsating mass on the upper middle abdomen.
The patient has black, tarry stools and anemia
The patient has had an endoscopic procedure two days prior
Submit

There is a pulsating mass on the upper middle abdomen.


Correct – This is an indication of a life-threatening aortic aneurysm. Palpating or percussing is
dangerous to the patient’s life.
A nurse understands that which of these patients are at risk for developing Oral Candidiasis, a type of
stomatitis?
A 77-year old woman in a long-term care facility taking an antibiotic
A 35-year old man who has had HIV for 6 years
A 40-year old man who is undergoing chemotherapy
An 80-year old woman with dentures
Submit

A 77-year old woman in a long-term care facility taking an antibiotic


Correct – This patient has the most risk factors for developing Candidiasis. Candidiasis is caused most
commonly by long-term antibiotic therapy, immunosupressive therapy (chemotherapy, radiation, or
corticosteroids), older age, living in a long-term care facility, diabetes, having dentures, and poor oral
hygiene.
What nursing intervention demonstrates that the nurse understands the priority nursing diagnosis when
caring for oral cancer patients with extensive tumor involvement and/or a high amount of secretions?
The nurse uses a pen pad to communicate with the patient
The nurse provides oral care every 2 hours
The nurse listens for bowel sounds every 4 hours.
The nurse suctions as needed and elevates the head of the bed
Submit

Correct – This intervention is in response to Ineffective Airway Clearance, which is the priority
nursing diagnosis.
A patient has been taking a mood stabilizing medication, but is afraid of needles. They ask the nurse
what medication would NOT require regular lab testing. What is the nurse’s best response?
Valproic Acid (Depakote)
Clozapine (Clozaril)
Lithium
Risperidone (Risperdal)
Submit

Risperidone (Risperdal)
Correct – Risperidone is the only drug that does not require blood draws.
A patient is deciding whether they should take the live influenza vaccine (nasal spray), or the
inactivated influenza vaccine (shot). The nurse reviews the client’s history. Which condition would
NOT contraindicate the nasal (live vaccine) route of administration?
The patient takes long-term corticosteroids
The patient is not feeling well today
The patient is 55 years old
The patient has young children
Submit

The patient has young children


Correct – This is not a contraindication. It would only be a contraindication for the live vaccine if the
young children were immunocompromised, but this is not stated.
A patient asks the nurse whether he is a good candidate to use a CPAP machine. The nurse reviews the
client’s history. Which condition would contraindicate the use of a CPAP machine?
The patient is in the late-stage of dementia.
The patient has a history of bronchitis
The patient has had suicidal gestures/attempts in the past
The patient is on beta-blockers
Submit

The patient is in the late-stage of dementia.


Correct – Having an inability to follow commands and understand instructions independently is a
contraindication for a CPAP machine, which can only function correctly with proper installation and
use.
The nurse is caring for a patient who has recently had a successful catheter ablation. Which
assessment finding demonstrates a successful outcome of this procedure?
The patient is free of electrolyte imbalances
The patient’s WBC count is within normal limits
The patient’s EKG reading is regular
The patient’s urine output is 45mL/hour
Submit

The patient’s EKG reading is regular


Correct – A catheter ablation is a procedure used to treat arrhythmias, especially SVT. A catheter is
inserted through the femoral vein or artery, and threaded to the conduction fiber in the heart causing
the arrhythmia. A radiofrequency energy uses heat to destroy this fiber, preventing further arrhythmia.
Application – The nurse is caring for a patient who has the following labs: Creatinine 2.5mg/dL, WBC
11,000 cells/mL, and Hemoglobin of 12 g/dL. Based on this information, which of these orders would
the nurse question?
Administer 30 Units of Lantus Daily
CT of the spine with contrast
X-ray of the abdomen and chest
Administer heparin subcutaneous 5,000 Units every 12 hours
Submit

CT of the spine with contrast


Correct – The creatinine level of this patient indicates impaired kidney function. Contrast is
nephrotoxic and is contraindicated for patients with nephropathy.
Application – A nurse is caring for a patient admitted in the emergency room for an ischemic stroke
with marked functional deficits. The physician is considering the use of fibrinolytic therapy with TPA
(tissue plasminogen activator). Which history-gathering question would not be important for the nurse
to ask?
“What time was the first time you noticed symptoms appearing consistently?”
“Have you been taking any blood thinners like heparin, lovenox, or warfarin?”
“Have you had another stroke or head trauma in the previous 3 months?”
“Have you had any blood transfusions within the previous year?”
Submit

“Have you had any blood transfusions within the previous year?”
Correct – This is not a relevant question and would not affect the decision to use TPA
A patient is being discharged from the med-surgical unit. The patient has a history of gastritis. The
nurse questions the patient on his usual routine at home. Which of these statements would alert the
nurse that additional teaching is required?
“I avoid NSAIDS. I only take a daily aspirin for my heart health.”
“I always avoid eating hot and spicy foods”
“I will continue taking my antacids with or immediately after meals”
“I will only drink coffee once a week, if even that often.”
Submit

“I avoid NSAIDS. I only take a daily aspirin for my heart health.”


Correct – Aspirin is classified as an NSAID and can exacerbate already existing stomach problems.
Aspirin should be avoided just like any NSAID for patients with gastritis.
A nurse is meeting a patient in their home. The patient has been taking Naproxen for back pain. Which
statement made by the patient most indicates that the nurse needs to contact the physician?
“I get an upset stomach if I don’t take Naproxen with my meals.”
“My back pain right now is about a 3/10.”
“I get occasional headaches since taking Naproxen”
“I have ringing in my ears.”
Submit

“I have ringing in my ears.”


Correct – This is a severe adverse effect of Naproxen and should be reported immediately since it may
indicate toxicity.
The nurse is doing an intake screening for a patient with hypertension. They have been taking ramapril
for 4 weeks. Which statement made by the patient would be most important for the nurse to pass on to
the physician?
“I get dizzy when I get out of bed.”
“I’m urinating much more than I used to.”
“I’ve been running on the treadmill 10 minutes each day.”
“I can’t get rid of this cough.”
Submit

“I can’t get rid of this cough.”


Correct – A common adverse effect of ACE inhibitors is a persistent, dry cough. A medication change
to another class of antihypertensives, like an ARB, may be needed
The nurse in the emergency room sees a patient who has been abusing alprazolam (Xanax). The
patient reports that he suddenly stopped taking Xanax about 24 hours ago. He presents with a visible
tremor, is pacing, expresses fear, and has impaired concentration and memory. Which of these
intervention takes priority?
Have the patient lie down on a stretcher with bedrails up
Give the patient a cup of water to drink and a small amount of food
Assure the patient that he will be okay
Alert the physician that the patient needs Xanax
Submit

Have the patient lie down on a stretcher with bedrails up


Correct – The 1-4 day period after Xanax withdrawal is the most dangerous. Xanax is a
benzodiazepine and withdrawal symptoms include life-threatening seizures. Having the patient lie
down with bedrails up is part of seizure precautions and is the first priority
A nurse cares for a child that is diagnosed with Hepatitis A. Which of these following precautions
would be most important to take to prevent transmission of this infectious disease?
Encourage the Hepatitis A vaccine for family members and siblings
Use needleless systems if possible, otherwise use careful needle precautionary measures
Teach the child and enforce strict and frequent hand washing
Teach the child and family the dangers of contaminated food and water
Submit

Teach the child and enforce strict and frequent hand washing
Correct – Hand washing is the single most effective way to prevent transmission of Hepatitis A.
Hepatitis A is a virus transmitted via the oral-fecal route and lives on human hands.
A nurse is treating a patient suspected to have Hepatitis. The nurse notes on assessment that the
patient’s eyes are yellow-tinged. Which of these diagnostic results would further assist in confirming
this diagnosis?
Decreased serum Bilirubin
Elevated serum ALT levels
Low RBC and Hemoglobin with increased WBCs
Increased Blood Urea Nitrogen level
Submit

Elevated serum ALT levels


Correct – ALT is a liver enzyme, and hepatitis is a liver disease. Elevated liver enzymes will often
signal liver damage.
Which of these patients would the nurse suspect as having the greatest risk of contracting Hepatitis B?
A sexually active 45-year old man who has Type 1 Diabetes
A 75-year old woman who lives in a crowded nursing home
A child who lives in a country with poor sanitation and hygiene standards
A sexually active 23-year old man who works in a hospital
Submit

A sexually active 23-year old man who works in a hospital


Correct – This person is both sexually active and works in a healthcare environment.
The nurse calculates the IV flow rate of a patient receiving lactated ringer’s solution. The patient is to
receive 2000mL of Lactated Ringer’s over 36 hours. The IV infusion set has a drop factor of 15 drops
per milliliter. The nurse should set the IV to deliver how many drops per minute?
8
10
14
18
Submit

14
Correct – Drops Per Minute = Milliliters x Drop Factor / Time in Minutes
The nurse calculates the IV flow rate of a patient receiving an antibiotic. The patient is to receive
100mL of the antibiotic over 30 minutes. The IV infusion set has a drop factor of 10 drops per
milliliter. The nurse should set the IV to deliver how many drops per minute?
11
19
26
33
Submit

33
Correct – Drops Per Minute = Milliliters x Drop Factor / Time in Minutes
Which of the following statements made by a client during an individual therapy session would the
nurse most identify as reflecting schizoaffective disorder?
“I just want to stab myself with this pen.”
“What’s the point in life anyways?”
“My thoughts are racing because of the conspiracies against me.”
“I hear voices every day and sometimes see old friends that don’t exist.”
Submit

“My thoughts are racing because of the conspiracies against me.”


Correct – Schizoaffective disorder is characterized by the mania and depression of bipolar disorder
with the delusions/disturbed thought process of schizophrenia. Racing thought are a classic symptom
of a manic episode, while conspiracies indicate paranoia.

Das könnte Ihnen auch gefallen